You are on page 1of 252

\

\: t

'\ \
- \
Q+r

I
t
t
i F or P edonnance M easurement
j
L-.

ZIMBABWEsc!{ooLEXAMINAT|oNScoUNclL

ED u cArroN
c EN E RAL c
mrfrfsrFEft[

MATHEMATICS
4008r4028

Expected Answers
Past Question Papers and

1996 - 1998
l

I
l

ouncil
C
Zimbabwe School Examinations
Oast Road, Mount Pleasant'
Harare
ippcr

@ Zimsec 1999

a relrieval Sysrcm,
or transmittetl in any form or
reProducet], storetl in
Atl of
rightsreserved. No part this.p*li,:,:!:^y,!-be
*n"'*i" niiio'u' priir pennission of the copyright owner'
pnorocopying' re["''dn'e "
by any neans, electronic, meinanriat,

.-*
'rl
t
,, f,
ti)

FOREWORD
The need for effective examination study booklets has been a continuing concern in
our
Zimbabwean schools. Due to a significant number of candidates who fail to Lome
up with five
'O' Levels at one sining, many teachers are forced to look for appropriate
examin.iion guides
in order to adequately prepare their candidates fbr the examination. Needless to
say the main
cause of underachievement as indicated in examination reports points to failure
by candidates
to understand and interpret the requirements of the questions. Teachers are handicapped
in
developing good examination techniques within their candidates as they do not have
relevant and
viable examination booklets. The rnajority of the study guides on offer are not relevant
ro the
Zimbabwean syllabuses that are currently being taugltt. The Zimbabwe School Examinations
Council (ZIMSEC) therefore has come up with ttie idea of presenting relevant and
viable
examination guides which focus on the syllabuses that are currently taughi. These
examination
guid91 present questions and suggested solutions. The aim of the guidesls to acquaint .O,
Level
candidates with the structure of the examination, questionmna expected solutions.
The
suggested solutions are meant to develop, within the candidates, effective examination tectrniques
and strategies relevant to the examination.

These booklets come in a series covering a number of subjects on offer at our examinations
centres. We hope that the series will contribute in developing, within our candidates, effective
examination techniques in the area of subject mastery, question interpretation and presentation
of answers. This service by the Council will benefit all our stakeholders whose main interest
is to improve the performance of our candidates in examinations.

(^^*r".^*
Dr I.M. Sibanda
DIRECTOR - ZIMBABWE SCHOOI- EXAMINATIONS COUNCIL
(ii)

lJ-rl :

ABOUT THIS BOOK

The book is intended to assist candidates preparing to sit the ZGCE'O' Level examination in
Mathematic s (40f,8 / 4028) .

Material for three years, 1996 to 1998 covering both the June and November examinations has
been put together. The candidate thus has an access to six examinations which should assist in
the practice.

The format of the papers and the depth of topic coverage will also be evident.

The solutions and Marking Scheme appear from pages 177 and are arranged in the order of the
Question Papers beginning with June 1996 Paper I and ending with November 1998 paper 2.

The section on "Expected Answers" focuses on numerical answers to most questions. They TIM
serve to give a ready check to the work of the candidate.
Methods for solving have not been included since there are several valid approaches to solving INS
a given problem.
Wri
Accuracy of answers varies from 3 significant figures to 4 significant figures to cater for the Anl
calculator version. Where constructions are involved, line segments to be measured have a Wr
range of values to accommodate for errors. lfv
Or
Mi

IN

TI
Candidate
Centre Number Number

landidate Name

UNlVEFISITY OF CAMBRIDGE LOCAL EXAMINATIONS SYNDICATE


nln in collaboration with
THE MINISTRY OF EDUCATION AND CULTURE, ZIMBABWE
Genera! Certificate of Education Ordinary !.-eve!
has
tin MATHEMATICS 400911 , 402811
PAPER 1

Wednesday 29 MAY 1996 Morning 2 hours 30 minutes

Candidates answer on the question paper


he Additional materials:
|.. Geometrical instrumenls

:y TIME 2 hours 30 minutes

rg
INSTRUCTIONS TO CANDIDATES
Write your name, Centre number and candidate number in the spaces at the top of this page.
Answer all questions.
Write your answers in the spaces provided on the question paper'
lf working is needed lor any question it must be shown below that question.
Omission of essential working will result in loss of marks.
Mathematical tables, slide rutes or calculators may not be brought into the examination room.

INFORMATION FOR CANDIDATES


The number of rnarks is given in brackets [ ] at the end of each question or part question"

FOH EXAMINER'S USE

This question paper consists of 16 printed pages.


MFK (0018) QF66986
,-er MOEC Zimbabwe
1996 [Turn over

..i'.4
NEITHER MATHEMATICAL TABLES NOR SLIDE RULES NOR CALCULATORS
MAY BE USED IN THIS PAPER

Express

(a) lQ{,709 correct ro 2 significanr figures,


(b) .7.6 x 10-3 as a decimal,
o
(c) a percentage.
TOas

Answer (a)
[1]
(b)
tll
(c)
trl

(a) Write 12 43 as a time on rhe l2-hour clock.

(b) Joan made a toy for her little brother.


On Friday she spent I hour 40 minutes on it.
On Saturday she spent 2 hgurs 35 minutes on it.
How long did she spend on it altogether?
,.1 -u
(1c)Je tra.in arrived at its destination at 0l 20 after a journey lasting 3 hours 45 minutes.
what time did the journey start?

Answer (a)
tri
(b) hours minutcs lrl
(c) Irl

{rrH,l r4r}28r l.,srxr

2
--
r \'
,,,T) \
RS simplify. giving your answer as a common fraction in its lowest terms,

(a) +l - li.
(b) 11+ z]r.

Each member of a group of 60 girls was asked what


was her favourite drink.
Their replies are represented on the pie chart'

(a) 24,girls said they preferred Orange. Calculate


ihe value of .r.

(b) Find the number who said they preferred


Lemon.

(c) Find the percentage of the group who said they


preferred Lime.

{IH/l'{.tlttr I rsql [Turn over

3
l:tortiter\ E
5 {a{ Express rhe ratio 1,8 kg : 400 g in its simplest form.
{h,

\-J

{tul }udith and Sibongile share a sum of money in the ratio 5 : 3 respectively. If Sibongile
\uJgets
#
$42, calculati the amount shared.

Answer (a)
ttl
(b) $
I2l

6l
----_--j i In the diagram, PSand eR arestraight
Iines and PQ is parallel to RS.
Given eSR = 100o, pflg = 61.
^that
and RQS - 27o, find the value o1
(a) t.

(b) m,

(c) n.

Answer (a) I = [1]


'> "o'
;-
.-
-/-
:?
(b)m= trI
(c)n= fll

{rt!,la{,2tll /s96

l-
,:.
(a) Factorise completely I + :i - tO,

fiilrira rhe towest common multiple (LCM) of the expressionsx2 + 3r - 10 arid (x - 2)2.

Answer

E (a) Express l0l r,, as a number in base 5.

f,? llvaluatc 11001, - I1102.


f(c) j Write down the values of N such that
\--l

Nbo."2=Nbuse5'

{IM/4t12U1696 JTurn over

5
9 Given that h=P3-q
l
make p the subject of the formula-

Answer P=

10 Given that logr2.;o,319and logr3 =9,5646,calculate the value of

(a) loq,.6.

(b) logr1,5,
,,
(c) logr8.

i
I
I
-1 {

t
I
t
1

I
I
i

i
I

I
I
I

I
,ffinlsllfitstl6

L
lttr
Etomincr's b
Usc 1
if,?
fu;'s
i

I I Solve the equations

(a) 3(5 -.r) = 2r.


/\r
: (h) (2.t' + 3)' = 49.
\-,
''.---..

IU

t2l

12 (a) Construct the locus of a point equi-


distant from the lines AB and BC.

(b) In the diagram, XYis a straight line.


Construct the locus of a point 2 cm
from the line XY.

x Y

.t(r Hi I /,fi,lra/l rlien [Trrn over

7
13 From the letters of the word CHARMS list the letters with

(a) no line of symmetry,

(b) one line of symmetry,

(c) rotational symmetry of order 2.

Answer (a)
t1I
(b)
[1]

(c)
l1l

14 The table below contains some information about Mr Dube's electricity bill for the month
of January.

Fixed charg@ = $15,00


reading
Initial meter =
Final meter reading . = w26
Number of units used = ..\.ii.........
Cost of I unit = 35 cents
Total amount payable = $58,40

Use the information in the table to find

(a) the number of units consumed,

(b) the initial meter reading.

Ansn,er (a) tzl


(b)
trl

.l(t)8,/ I /4r)ltt/ I is90

8
15 (a) Solve the inequalitY - 4 < 2x + L. ''

(b) List all the integer values ofx which satisfy the inequalities - 4 < 2x+ 1 and x <-2'

Answer (a) tu
(b) 121

l6 7.)
-4.1' o! rthnf ,1,1 rsl zgl "1113+,
!

Thc data above is arranged in numelical order'


I
(a) Find the value of /r such that the inedian of the data is 13.
(b) (sing the value of lr in (a) calculate rhe mean of the data.

Answer (a) h = l2l


(b) mean= IrI

.l(xwt/402lvllsir [Turn over

9
I
l

tatl-
grlrts't hrr'
art I '
t-
17 A shop owner allows a discount of 15"/o on all cash sales. calculate
l

(a) the cash price of a pair of shoes with a marked price of $90,fi).

(b) the marked price of a dress which was sold for $204.(x).

Answer (a) $ trl


(b) $ tzl

16 Study the pattern in the table below.

3x5 6xl = 9
4x6 -- 7x2 = l0
5x7 8x3 = ll
-t a .' o

'aaa

aa

(a) Write down the sixth line in the pattern.

(b) Find positive integersp and q such that

px14-15xe=ltl.

Answer (a) trl


(b)p=
q- I2l

.llXIrl/$2tltrS(5

l0
l9 Given that p = 4. evaluatc

(a) (zpli.
(lr) tt/r-'.

Attstvcr (tl) I2l


(b) I2l

Bo

In the cliagram, ABC is a triangle with AD perpendicular to BC procluced.


AC: l7 cm, AD:8 cm and ABC:23,4".
Usr=ng as much of thd information given below as is ilecessary,
.^
(a) write down the value of sin AQp,
(b) calculate
(i) DC.
(iil Au.
[sin 23.4'= 0.4(]l . cos 23.4e = 0.92: tan23,4" = 0,43I

Attslver' (a/ sin ACB = lrl


(b) (i't DC = Irl
(ii) AB = Izl

. {rr.$ru4l,2|l^tft [Turn over

lt
GiventhatA=(2 3), B = (_ -
f),,..= (? 3), o"o
(a) c2,

(b) BA.

22 Express 2x 3+x
3- 6 as a single fraction in its simplest form.

Answer
{41

il{ntll/402fi/t1596

t2
(n) A regular polygon has r sides. Write down, in terms of n. the size in degrdes of
(i) the exterior angle of the polygon.
(ii) the interior angle of the polygon.
(b) The difference betrveen the interior angle and the exterior angle of a legular polygon
is 100.. Given that the interior angle is greater than the exteriorangle', find the number
of sides of the polygon.

(r) Given that r 5 < .r s 3 and.r is an integer, find

(i) tHe nlinimum value of .r'2.


(ii) the maximum value of 2r -.-r'1.
(b) The lengths of the sides of a triangle. measured to the nearest centimetre. are t I cm.
8 cm and 6 cm. Find the minimum perimeter of the triangle'

{rx}uu{r):r( l.ss [Turn over

r3

,@
25 The diagram shows the graph of a straight line / and
points P and C.

(a) Find
(i) the gradient of the line /,
(ii) the equation of the line parallel to I which
passes through the point P.

(b) The point P is mappecl onto another point e by a


clockwise rotation of 90'about C.
Find the coordinates of Q.

\\
\
l
\
i
An.swer (a) (i) trl
l

(ii) [21

[21

26 A bag contains 10 identical balls of which 3 are red, I is blue and 6 are green. Two balls are
drawn, at random, from the bag without replacement, one after the other. Calcutate the
probability that

(a) the first ball is green and the second is blue.

(b) both balls are blue,

(c) both balls are of the same colour.

40{ni/t /4028/l /s96

l4
E
,-
F*,
tb
line / and
The volume v of agas varies directly as the temperature Iand inversely as the pressure P'

(r) Find a formula for V in terms of T, P and a constant


k'

k'
O) Giventhat V= l0when T=ffiandP=8'findthevalueof
r/ which
(c) Calculate the value of V when T = 150 "n6 p = 40'

rtQbya

. trl
. t2l
| {21

ls are
e the

Answer (a) t2l

(b) 12)

(c) tu

.1(Xl8/ li{2& l/s(r6


[Turn over

t5
for
tittminet's
L.'sc

?a

I
t,
I

,---

The diagram shows a rectangular block of wood measuring.r cm by x cm by 5 cm.

(a) Find, in terms of x, the total surface area of the block.

(b) Given that the total surface area of the block is 400 cm2, find the value of x. Hence
calculate the volume of the block.
x
WI
pe
N
w
fr
AI
o
r
tr

I
T
I

l2l
I2l
tll

l(xpt/{Dtll/sqr

l6
\

UNIVERSITY OF CAMBRIDGE LOCAL EXAMINATIONS SYNDICATE


in collaboration with
THEMINISTRYoFEDUcATtoNANDCULTURE,ZIMBABWE
General Certificate of Education Ordinary Level
MATHEMATICS 402812
PAPER 2
WednesdaY 5 JUNE 1996 Morning 2 hours 30 minutes

Additional materials:
Answer PaPer
GraPh PaPer (2 sheets)
Mathematical tables
Plain PaPer (1 sheet)

Tre 2 hours 30 minutes


{ence
ffiiTRUCTIONS TO CANDIDATES

ffiae your name, centre number and candidate number in the spaces provided on the
answer

ffir/answer booklet'
B'
In5lner all questions in Section A and any three questions from Section
trnte your answers on the separate answer paper provided'
F pu use more than one sheet of paper, fasten the sheets
together.

fl working must be clearly shown. lt should beofdone on the same sheet as the rest of the answer'
&rission 6f essential working will result in loss marks.
the answer is not exact' the answer
f tfre degree of accuracy is not specified in the question and ifshould be given to one decimalplace'
$m.dd be given to thiee signiticani figures. Answers in degrees

I5ORMATION FOR CANDIDATES


question or part question'
Ttre number of marks is given in brackets [ ] at the end of each
numericalexpressions.
ffirematical tabres or electronic carcurators may be used to evaluate expricit

r.z)

2l pilnied pages and 1 blank Page.


@onsistsofl
sB (cE) oF67001 [Turn over
I 6 MOEC Zimbabwe 1996

t7

.j
Section A [64 marksl

Answer all the qucstions in this section"

(a) Given that a = 2 and b = -3,calculate


(i) &- b2,

(ii) (a - b)2,
(iii) +
ND

(b Solve the equation

,1 *l=?.
2
3

Th. values of x and y are such that


tt}
x+3Y=J,
i, **.=:t"=.5'
O Factorise
x2 -'gyz
and hence, without solving the above equations, write down the numerical value
orl -gf .

(O Solve the simultaneous equations


x+3y=3,
x-3Y=J.

(a) A tourist in Zimbabwe cashed an f,80 cheque at the official exchange rate of 2$12,85 for
11 sterling
(i) Calculate how many Zimbabwean dollars he received for f80'
(ii) He then bought a camera f.or Z$578,2.5. Calculate the equivalent cost, in f sterling, of the
camera'
t3r
O) Ir ir given that c is an integer and that
'E={c'.1 <c<10i,
P = lc: c is even),
g = lc c is a perfect cube).
A.?
(i) List the elemenrs of ^'{ '

.rL d-nl (-
'
(al Q, \/ ,a
^(\
(b) PaQ,
(c) (P \r q)'. ,

(ii) Write down the value of n(P t-' (J).

18
!
1

i,,, ln rlrc cliagrarn t\,. B. C and D are points on tlelcl-:umfe^rence of


-l'hc
chord.s llCand CD are equal in length and CIR =
l8o'
a circle with dianleter AB'

(i) E,xplain brieflv whY D,4C = 2tlo.


(ii) Calculate (a) A6C. i
(b) AcD.
[4]

[., O) Answer this parf of the question on the plain paper provided'
arcs must be clearlv shown'
use ruler and compasses onlv. All construction lines and
(i) Construct. in a single diagram'
(a) the triangle PQR inwhich PQ = l}cm' QPR = 45" and PfR = 30''
(b) the positiorr of the point s which is equidistant from P and Q and also from Q and R.

(c) the circle passing through P' Q and R'


(ii)Measureandwriteclowntheradiusofthiscircle. rtl
I

f6l

I'furn over

l9
In this quesrion rake r tobe?Z,
7-
In the diagram, oyz isa sector of a circre, centre o and radius 9 cm.
Given that y6z = 45,50 and that x is the point on olz such that oiz= g00, catcutare
(a) ox,
12l
(b) the length ot arc yZ,
l3l
(c) the area ofthe shaded region.
l6t

N
In thc rliagranr. 1rQ'f = f.in..CUt = 3 cnr. !r'l'= 4cnr. 7p = 6 cm and pe= 5 cm.
(i) Nantc. in correct orclc'i:'the tso triiingles that are similar and explain why briefly.
(ii) Clalculatc 7'.S.

t.5I

(b) (i) Given rhat c = (- antr d = (_;)


i)
(s) express 2c * 3d as h colunrn vcctor.
l2l (b) findldl.
(ii1
I3l

l6l

11
!e tllagram, il =p and ,Th = q.M is rhe mid-point of AB and X is the mid-point
of OM.
Elpress as simply as possible in terms of p and q
(r) ./I}.
*
s(b) xM.

[Turn over

2t

,",i
(a) Find the value of .r for which the matrix is singular.
I i:
1

(b)

The sketch shows part of the curve I = 4 + 3x - x2 '

(i) The curve passes through the point P on the y-axis. Write down the coorclinates of P.
1ii)
*' The point Q on the curve has coordinates (p..g). Find the value of r7.
i l\l
(iii) The curve passes through the points A and-B on-the x-aiis. Calculatc the coordinatcs 9l
(a) A.
(b) B,

(iv) At the point R the.gradient of the c.grve is zero..Calculate the


;l-coorclinate
of
' R.i\
i tltl

22
i--
12l Section B [36 marks]

Answer three questions in this section.

In the'diagryfi, C is due east of ^rl and ACD is a straight line.


u
AB = 6km, BC = 7 km, CD = 10 km and Be D = 135".

Calculate.

(r) BAC, t3rtf

O) the bearing of B from .zt correct to the nearest degree, plt


(c) the shortest distance from B to AC, tzl
(d) BD. tsl

I'I'urn over

tr

F,
.r ll (a) Stllve thc equation x2 + 2x - l0 = 0, giving your answers c()rrcct to 2 decimal placcs.

,"(!)

36

Speed
(km/h)

Time (minutes)

The diagram is a speed-time graph for part of a train journey.

At a distance of 2,5 km from train retards uniformly from a speed of V.km/h and
a station, the
stops at the station after 10 minutes. It remains at the station for a further 10 mintites and then
accelerates uniformly to a speed of 36 km/h in 5 minutes.

Calculate
(i) the value of V,
(ii) the acceleration, in m/s2, of the train in the last 5 minutes,
(iii) the average speed, in km/h, of the train over the total period.
l8l

24
I
l

Answer lhc whole of this qucstion on a shcct of graph paper.


t4l
A farmer supplics a shop with eggs which are gradcd either medium or large. In any week the
grcatest number o[ eggs that thc shop requires is 60 dozen of which x dozen are medium and
.t'dozen are largc.

(a) Writc dorvn an inequality which cxpresses this condition.


IU
In any wce k thc numhcr of dozcn of medium eggs that the shop requires is at teast twice the number
of dozen of largc eges.

(b) Writc down an incquality which expresscs this condition. t2l


'l'hc shop pays the farmcr
$7,00 a dozen for medium cggs and $7,50 a dozen for large eggs. In any
wcck thc shop pays the farmer at least $210.

(c) Writc clown an incquality which satislics this condition and show that it reduces to
[4r + 1-5.y'> 42A. Izl
(d) 'l'hc point (.r,
-v) rcpresents .r dozen medium eggs and -r' dozen large eggs. Using a scale of 2 cm
to represcnt l0 dozen on each axis, cclnstruct and indicate clearly by shading the unwented
rcgions, the region in which (x, y) must lie. t4l
(e) [Jse your graph to find, in any week,
(i) the lcast number of dozen of medium eggs that the farmer could have supplied to the shop,
h and (ii) thc maximum amount thc shop could have paid to the farmer. 13l
I then

l0 [ln this question takc ti to be 3,142]


'lhe water supply tor Chcngwe Day Secondary School is
contained in a closed cylindrical tank of
intemal radius 110 cm and internal height 345 cm.

(a) Calculate
l8l
(i) the volume, in litres, of the tank, giving your answer correct to the nearest hundred
litres, t3l
(ii) the total intcrnal surface area, in m2, of the tank. t3]
'lhe tank stands with its
circular basc on horizontal ground.
When thc water sinks to a level of 30 cm above the base, a pump switches on to refill the tank and
switches off whenit is full.

(b) Assuming that no water is drawn from the tank whilst the pump is filling it up, calculare the
volumc, in litrcs, of watcr pumped into thc tank cach timc it is refillcd.
lzl
(c) It costs 70c pe r hour to run the pump. During a ccrtain wcck thc tank was refilted 20 times at a
total cost of $3.5.
('alcutatc thc ratc, in litres per hour, at which the water was being pumped into the tank. t4l

['furn over

L5
II

!-

J
.100

o
r 300
o

i)

,L)
!
E
z

60
Marks scored

The graph is a cumulative frequency curve showing the marks scored by 500 candidates in a

mid-year Mathematics examination.

It is given that
6A0/o of the candidates passed the examination.
a candidate who scored more than 75 marks is classified as grade A,
a candidate who scored more than 60 marks but less than or equal to 75 marks is classified as

grade B.

Use the eraph to estimate as accurately as possible

(a) the rnedian mark, [1]

(b) the pass mark. 121

(c) the prercentage of candidates classifred as


(i) grade A.
(ii) grade B.
[4]

(d) the probability that one candidate sel,ected at random scored at most 65 marks. t2)

26

!
\

(r)
Marks scored 0<r<20 20<.r<.50 50 <.r < 80 ll0<x<100
iio +b5 $to
Numbcr ol
30 r80 -v Z
candidatcs ,- f'

'l'his tablc also shows the <Iistribution of the marks of the ,500 candidates, but in a different
lilrnr.
Find the valuc ol y and the value of z. t1I

(f) 'l'his information was illustrated by a histogram and thc column representing 0 < x < 20 was
2 cm wide and 1,-5 cm high.

For the column reprcsenting20 < x < 50, find


(i) the width,
(ii) the height.

Answer the whole of this question on a sheet of graph Paper.

.Triangle X has its vertices at the points (3,2), (5,2) and (5, 4).

(r) Using a scale of 2 cm to represent 2 units on each axis, draw axes for values of x and y in the
ranges - 4 < x < 12 and - 6 < -t' < 14.
Draw and label triangle X. \ t1I
tat .,14
2.-
-'
ina pi lrre transformation 'I' is the translation ( i) td(J- {
I

Draw and label the triangle T(X), the image of X under T. tzl

G) The transformation P is a reflection in the x-axis.


Draw and label the triangle P(X), the image of X under P. t1l

(I) 'l'riangle Q(X), the imagc of X under transformation Q, has vertices at (- 1, - 2), (- 3,-2)
and (- 3, - 4)
(i) .I)raw and label triangle O(f')
(ii) I)escribc fully thc sinqle transformation Q.
[1] t3l
I2l (c) 'I'rianglc
O(X) can be mapped onto triangle l'(X) by a reflcction.
Find the cquation of the mirror linc. tlI
(f} 'l'riangle R(X), thc image ol' X under transformation R, has vcrtices at (6,01, (10,6)
and (10. l2).
141
(i) I)raw and labcl trianglc R(,t)
121 (ii) I)cscrihc fully the transformation R.
I4l

27
Candidate
Centre Number Number

hoate Name

UNIVERSITY OF CAMBRIDGE LOCAL EXAMINATIONS SYNDICATE


in collaboration with
ZIMBABWE SCHOOL EXAMINATIONS COUNCIL
General Certif icate of Ed ucation. Ordinary lovel

MATHEMANCS 4008/1 ,402811


PAPER 1

wednesday 13 NOVEMBER 1996 2 hours 30 minutes

Candidates answer on the question paper.


No additional materials are required.

TIE 2 hours 30 minutes

rcTRTrcTloNS TO CAI{DIDATES
re your name, Centre number and candidate number in the spaces at the top of this page.
all questions.
'#rt your answers in lhe spaces provided on the question paper.
It rorking is needed for any question it must be shown in the spaoe below that question.
Crrltssion ol essentialworking will result in loss of marks.
Idpmatical tables, slide rules and eleclronic calculators may not be brought into the
rcrnination room.

IfORi,|ATION FOR CANOIDATES


Tl,te numberol marks is given in brackets I Iat the end ol each question or part question.

FOR EXAiTINER'S USE

5K (02r3) OK681Sl
c zsEc 1996 [Turn ovcr
29
For I
Fmincr'sl
Itsc I

NEITHER MATHEMATICAL TABLES NOR SLIDE RULES NOR CALCULATORS


MAY BE USED IN THIS PAPT:R.
a
Calculate the exact value of

(e) 21, iu,

tul r4,g -z,ts.


(c) t6@.

2 Factorise completely

(a) ty' + xzy,

(b) 98-U2.

lrl

12I

.(Dtrlrwir

30

;
(e) Express
(i) 7,369 correct to two significant figures,

(ii) * as a decinral. giving r-our answer correct to two decimal places.

(b) Evaluate
2 x l0r - 8 x l0:, giving vour ans$'('r in standard form.

Answ'er (a) (i) tll

(ii ) rir

(b) lrI

(e) Find the lowest common multiple (L.C.M.) of

3nt2n3 and 5m3n.

(b) Given that f(x) = d.rr + 5.r - 6, find the value of a for which f(- 2) = tt.

-,]
i, ;'

,lnstr'icr (a ) lrI

l(rN I \A qrr
ITurn over

3l
lu
Etnina
llv Fs
&-th
A plan of a house is drawn to a scale of 1 : 50'
the actual length of the room,
(e) On the plan, a room has a length of 8,4 cm. Calculate
giving Your answer in metres'
represented on the plan by an area
(b) Calculate the area, in square metres, of the room
of 63 cmz.

In the diagram, PQRisa right-angled- triangle T!


S]t a point on the side PO' It is given
itr.i pS =-18.r, 9S =ZZcm, gR = 41 cm and PR = 9 cm'
in its lowest terms' find
Giving each of your answers as a common fraction

(e) sin SOR, (b) css PfrQ, (c) tan OSR.

32
lrm,
Itisgiventhat -8<m<6 and -9<a<5.
Find (r) the greatest possible vatue of

[ea lil mn,


(iil m - n,

(b) the least value of m2 + n2.

tu

l2l

t h this question take n to be f.


ln the diagram, POQis a sector of a '
circle, @ntre O and radius 3,5 cm.

Given thar. PAQ= 36o,

(e) write down the ratio of the lengths of


the major arc PQ tothe minor arc PQ,

o) calculate the area of the sector POQ.


P

.#-lcro
<)to

Answer (a) ratio =

(D) Area of sector = ............ crl? tzl

aGrlrurr6 [Turn ovcr

33
Fa
&'an*t't
lh
In the diagram, A, B, C ahd D are
c
points on centre o,with Ac
a circle,
as the diameter. "4I is a tangent to
the circle. 35"

Given thal Ae B = 35o, calculate

(a) ADB,

(b) A6c, o

(c) air.
,/
U

tu

l0 (a) Calculate the value of 10002 -' l2 ' leaving your answer in base two'
(b) Write 5r'r 5 as a number in base five.

(c) Convert 34, to a number in base two.

tr I

Irl

lqpuwqi

34
Iot
figrnilut's
l:*
'l 6i can be'reduced to .r: + 3.r = 0.
(a) Show that the equation
.rt-l .r-l
-

l2l

(b) Solve the equation xr + 3r = 0'

Answer (b).r = or ...............'.. t?l


I

intersecting at 'Y'
In rhe diagram, PQRS is a rhombus with diagonrrts PR and QS

(iiven that PR = 30 cm ancl QS = l6 cm. citlculate

(u) PS,
(b) the area of the rhombus P0RS'

tGIg.lrWct.
[Turn over

35,

0t
Fot
F.tofu't
Ut
13 The acceleration a of a particle moving in a circle varies jointly as the square of the
velocity v and the inverse of the radius r of the circle.

(e) Write down an equation that expresses a in terms of u, r and a constant *.

(b) Given that the acceleration of the particle is 20 m/sj, when the radius is 1.8 m and
the velocity is 6 mls, calculate the value of the constant &.

(c) Calculate the velocity of the particle when the acceleration is 30 m/s2 and the radius
is 2,7 m.

(b)k= lrl

3r?
36
class interval frequency

35-39 1
N-U 3
I and 45-49 7
**--,"50:-54 t2
55-59 25
tdius ffi-64 38
65-69 23
70 -74 4
75 -- 79 1
80-84 I
\
The table shows the frequency distribution of marks scored by Form Four students in a
mid-year Mat hematics Examination.

(e) State the modalclass.

(b) Find the number of students who sat the examination.

(c) Calculate the percentage of students who scored marks less than or equal to 54.

trl

rl tt I

rl I2l

amrlr*t [Turn ovcr

37
F*
Etaniact's
Use

ls It is given that M =5#.


(a) Calculate, giving your answer as a common fraction in its lowest terms' the value of M
when ?n = -20.

(b) Express lin terms of M.

(b)

,l{IXtrl/Wc6
6517

38
fi tn this question use the exchange rates per Zimbabwean dollar (Z$) shown in the table.

rcaf M
Currency

I Ilritish Pound Sterling (f) t3,20 (z$)


I South African Rand (R) 2,40 (Z$)

A wonran left Hararc on a business trip to l]ritain and South Africa. Her company gave her
8)60 busincss allowancc.

(e) While in llritain she spent I of ttris allowance. How many British Pounds (f) did she
have left?

(b) On her arrival in South Africa she changed all the remaining allowance into rands.
Calculate thc amount she received in South African currency.

(c) On her return to Zimbabwe she had Rl05 left. She changed this to Zimbabwean
dollars. Clalculate the amount she received in Zimbabwean currency.

Answer (a) f tu

(b) R I2l
t2l

(c) z$ tt l

tzl

{If,,lAf,{r

39
t7 (r) Eraruarc (e ,(-3)

(b) tt isgiven rhat and r = (;)


" = (- ir)
(i) Find le l.
(ii) Given that C = 3r + 2b, find C.

trl

trl

,BIrSr
(ii) c =
o Izt

0
The diagram is a road sign which shows that it is 282 km to BINGA.

(e) Write down the letters in the name BINGA which have
(i) line symmetry.
(ii) rotational symmetry of order 2.

passing
Ot Mrs Moyo, travelling by car, takes 4 hours 42 minutes to reach BINGA after
the road sign.

Calculate, in kilometres per hour' her average speed'

tr I

tr I

Izt
01Um(i

4t
19 Evaluate

(a) (-le)",
(b) 3? x 23,

(c) (*) '

1.,
I

(c) I2l

.{DAl/WrXr

42
v

l9

rrtlcrity
in nretres
pcr second

The diagram is a vetocity-time graph of an object which retards uniformly from a velocity
of 19 m/i, to a velocity of tO mli in6 seconds. It then moves uniformly at this velocity for
8 seconds and is brought to rest in a further time of 15 seconds.

Calculate

(a) the retardation of the object during the first 6 seconds,

(b) the velocity of the object when , = 4,

(c) the distance travelled during the period from r = 6 to t = 29'

{UIVI/WI'(I [Turn over

43
t'd
Etollrdnal
Uv
2l (a) v

In the Venn diagram above

I is the set of Form One pupils,

I is the set of boys and

S is the set of pupils who play more than one sport.

It is given that there are 100 Form One pupils.

(i) write down the number of bop who play more than one sporr.
(ii) How many pupils are girls?

(ii) tll
(b)
a

Another Venn diagram shows the universalset I and sets p and e.


Shade, in the diagram, set pn e,. I2l

0uuw{6
65t7 6517

4
fot
Etuim't
Us

Evaluate
(i) logr.i\ 16,.
,.

log'o 27
,r,, .
log,6 3

Given that log,,, ? = 0,301- and log,o 7 = 0'&15' calculate the value of
(i) log,r 3,5,
(ii) 13g,,,4Q,

tll

tll

{{Ipt/w [Turn over

45
23

r) (.5 -.r)

In the diagram, the graph of y = (2 + x\ (5 - x) cuts the r-axis at A and B, and the y-axis
at C.

(a) Find the co-ordinates of .4, 'B and C.

(b) (i) Calculate the gradient of the line BC.


(ii) Hence, or otherwise, find the equation of the line BC.

(c) Find the equation of the line parallel to BC which passes through .4.

A nswe r fat 4..i1..(...................'..... ........'.... )

(b) (i) gradient = tl l

(ii) equation is ............ tll

(c,) equation is ............ tll

4
Fa
Etittl
Tb

In the diagram, AB is parallel to CD and EF is parallel to BC'

OB = EF = 7 cm, OE =6 cm and ED = 1'2 cm'

(e) Name, in correct order, a triangle in the diagram that is


(i) congruent to triangle DEF,
(ii) similar to triangle ABO.

O) Calculate the length of OC.


(c) Giventhattheareaof triangleABOis16cm2,calculatetheareaof trapezium OEFC"

Answer (a) (i) triangle t1l


t3I

(ii) triangle tu
tl I
t2I

t1I

t2l
tll

a{rruw96 [Turn over

47
as is necessary'
the informatio given b'elow
?s In this question use os much of

TIII

straight line'
rus
ln the diagram, QfrS =53", PR =
10 m' QR =8 m and PRS is a
Wri
AE
(a) Calculate
ArE
(i) the area of triangle PQR' w?i
form'
(iil PQ,leaving your answer as a surd in its simplest lfy
(b)ffi ff,'lftlilf,,Tl$l?f;t'#Jtr'f;ff 3;?ffi ',t'i;ff Elc
i',",",:T:lJ:';:f ':I,f AI,
tan 5f = 1,331. Or
cos 53o = 0,6&
[sin 53" = 0,8O ffr
be

'l.l
tt
il':
fiii
tl'
r

I'iii'l
lllr I I

iliil
'rii'll
il
Answer (a) (i) ...........................""""'
m' l2l
liti
:li
:'i
lrl
lr
m t3l
i.
iii.
rl
ii1:i m I2l
h.r,
;
I i, rF
c2
h:

6617
48
,1t

UNIVERSTTY OF CAMBRIDGE LOCAL EXAMINATIONS SYNDICATE


in collaboration with
ZIMBABWE SCHOOL EXAMINATIONS COUNCIL
Generat Certilicate of Education Ordinary Level
MATHEMATICS 400812
PAPER 2
Friday 15 NOVEMBER 1996 2 hours 30 minutes
Additional materials:
Answer PaPer
Geometrical instruments
GraPh PaPer (3 sheets)
Mathematical tables
Plain Paper (1 sheet)

Iffi 2 hours 30 minutes

GTRUCTIONS TO CANDIDATES
Irib your name, Centre number and candidate number in the spaces provided on the answer paperl
G;er booklet.
trruer atl questions in Section A and any three questions lrom Section B.
trile your answers and working on the separate answer paper provided.
f yorl use more than one Sheet of paper, fasten the sheets together'
&clronic calculators must not be used.
fl:orking must be clearly shown. lt should be done on the same sheet as the rest of the answer.
Onssion of essentialworking will result in loss of marks.
tf,rc cbgree of accuracy is not specified in the question and if the answer is not exact, the answer should
D $iuen to three significant figures.

FOR CANDIDATES
-ORMATION
Tte ngmber of marks is given in brackets [ ] at the end of each question or part question.
ffirematicaltables may be used to evaluate explicit numericalexpressions.

This question paper of 11 printed pages and 1 blank page'


Iq(@14)QK681s2
czlEC 1996 [Turn over

49
Section A [64 marks]

Answer all the questions in this section'

I (a) Find the numerical value of

P(l-q')
I2l
whenP=4and Q=-3'
tu
(b) (i) Factorise x? +7x + 12'
l2l
(ii) Factorise completely km - kn - lm + ln'
I
t3l
(c) Express '+ + *a single fraction in its simplest form'

2 (a)

points A' B' C and D'


the centre of a circle which passes through the
. .\
BCE and CDF are straight lines'

8A is parallel to CD, e,f :80o and gfiA = 45"'


which are also 80"' t3l
(i) Name three other angles on the diagram

(ii) Fird
t3l
(e) reflex AAD,
I2l
&) A6'0.

6518

50
t

lr
t
P
t3l

In the diagram, PQRS is a trapezium in which PS is parallel to QR. The diagonals PR and QS
meet at Iso that QT =2 cm and IS = 1 cm.

Given that the area of triangle IRS is 6 cm2, find the area of
(i) triangle QPT, t21
(ii) triangle QPS. lzt

h\1
6) Aisthe*rt.i*/ 8 3\ *-I
[-'-ul
L. L.I J -t
(il Find
(a) the determinant of A, t1l
I

(b) the inverse of A, tzl


(c) A2) {21

(ii) Using your answer to (a) (i) (b), or otherwise, solve the simultaneous equations
+3/+1=0,
5e-2f+2=0. , t3t

}} g = {x: 30 <.r < 60 where x is an integer}.


' A- {r x is a multiple o(l.J}.
g = lx: r is a factor.of 2000).
C {x: x is the cube of a riumbei between 1'and S}.
=
D - lx:r is a prime number ).
(i) List the elements of the set t
(a) A, tu
,\(,
-a\ -
'i- r-l ,r
(b) 8. (1,' .- 4 tll
(ii) Find
t3l
(e) n (C), tlI
tzl
(b) n (D). trl

[Tunr over

5l
Answer the whole of this question on a sheet of plain paPer.
(a)
Use ruler and compasses only for all constructions and show clearly all the construction lines and arcs.

(a) Construct, on a single diagram,


(i) triangle VWX in which WX =9 cm, VfrX = 60o and VRW = 45", t3l
(ii) the locus of points which are equidistant from VW and VX, [21

(iii) the locus of points which are 2 cm from WX. t3l

(b) On your diagram for part (a)

(i) rrreasure and write down the length of VX, lrl ,(6)

(ii) mark and label two points A and B which are 2 cm f.rom WX and equidistant from
WV and VX. l2l
(c)

(a) A shop-owner made a loss of 12% when he sold an item for $264..
What was the cost price of the item? l3I

(b) In 1Q!5 the interest rate on mortgages was reduced from 32o/o p.a. to 30% p.a.
Find the difference in the monthly repayments on a mortgage loan of $48 000. t3l

(c)

In the diagram, A, B and C are points on level ground such that AB = BC. The bearing of
B from A is 040" and ABC = 140".
Find the bearing of
(i) C from B,
(ii) A from C.
t4l

6518
6518
(a)

[:

[:
ln the diagram RS?"is a triangle in which RS =3 cm, SZ= 4cm and RZ= 6cm.
Calculate RIS. t4l
(6). From a seat in a football stadium the angle of depression of a ball on the level ground
is 28". Given that the seat is 5 m above ground level, find the distance of the ball from
the seat. t3l
(c)

t3l

t3t

In this part of the question take ,r to be +.

ABCD is a rectangular metalsheet. It is bent, as shown in the diagram, so that the arcs AB and
;ring of DC are each semi-circles of radius 7 m.
AD=BC=10m.
Calculate
(i) the length. of the arc DC, I2l
(ii) the outer curved surface area ABCD. I2l

.{)fit/2/wq6
[Turn over

53
Section B [36 marks]

section'
Answer three questions in this
carries 12 marks'
Each question in this section

In this question teke r to be f'


and
wooden log of radius t4 cm
(n) The diagram shows a cylindrical
height 50 cm.
4
l4
Calculate
l2l
(i) the volume of the cylinder in cubic centimetres'
in square
(ii) the total surface area of the cylinder t4l
50 centimetres'
Int
(a)

30 cm
A right circular cone with base of radius 14 cm and height
(b) .14' is Or-itteO in the toP of the log'
Calculate
12]l
(i) the volume of the cone'
cone of radius r and height h
isLrnr2h.l
l,. [The volurne of a
12)
(ii) the slant height of the cone'
(b)

the volume oflle cone removed l2l


(c) Calculate thc ratio of ffierremaining
(c)

t 6518
100Br2/W96

6518

54
s14cm

o
l-

----->
--.>
diagram, OA = 5a, OB = 5b, BC = a - b and OD : DC =3: l.
Erpress, in terms of a and/or b, the vectors
_>
eight 30 BA,
_,>

AC,
--+
OC, 9
--> kY'
OD, .-V6 ,,- *t\
lr'r2n.1 -..-> \.) /
BD.

Gircn th at E is the mid-point of OA frnd, in terms of a and/or b,


O- EA,
_>
ET CE.

ar> ,l
G) What property is satisfied by the vectors CE and BD
State the reason.

c) Find the rutio-BD: CE in its simplest form.

{xlxut!6 [Tum over

55
l0A

of graPh Paper' T
Answer the whole of this question on a sheet
Board, bY small scale farmers in the
Hurudza L
The amount of maize sold to the Grain Marketing ra
area, is shown in the table below'

0<x<6 6<x<10 10<r< 15 15<x<20 20<x<26 26<r<30 (r

(l
Number of farmers

rmulative frequencf table for this distribution'

(c
Number of farmers who
sold this mass or less
l2l

the horizontal axis and a scale of 2 crn to


(b) Using a scale of 2 cm to represent 5 tonnes on for this
a smooth cumulative frequency curve
reprJsent 5 farmers on the vertical axis' draw (d
diitribution. Use your graph to estimate
(i) the median for this distribution,
or less'
(ii) the number of farmers who sold 24 tonnes t6l

probability that he sold 10 tonnes or less. l2l


(c) A farmer is chosen at random. carculate the
probability that each farmer sold
(d) Two farmers are chosen at random. calculate the I2l
morethanl0tonnesbutlessthanorequalto20tonnes.

i
651 8

4l,)vrwrx!
6518
56
the whole of rhis question on e single sheet of graph paper.

ABC has its vertices at A(6, 4), B(7,6) and C(g,4).


Hurudza
a scale of 2 cm to represent 2 units on each axis, draw axes for the values of .r and y in the
0<-r<l0and0<v<16.
Draw and label triangle ABC. tll
AB S I is a triangle whose vertices are at A(6,4)', B,(4, 0) and Cr(0, 4).
(i) Draw and label triangle AB rC r.
(ii) Describe fully the single transformation which maps triangle ABC onto triangle
ABSr
t4I

A.B.C2is a triangle whose vertices are at Az(2,12), Br(4, 16) and Cz(8,12).
(r) Draw and label triangle AzBzC2.
I2l
1ii) Describe fully the single transformation which maps AB,Cr onto A28zA?
2cm to t4I
for this
Draw the image of triangle ABC under a one way stretch of factor lf with invariant
Iine y = Q.
Label it A 18 tC 1.

l6I

I2l

pt

{nV2/Wtx' [Turn over

57
t2
Answer the whole of ihis question on a single sheet of graph paPer.

A Co-operative wishes to go into the passenger transport business using station wagons and ntini
buses. Use the table below to answer the questions that follow.

Type of Cost of Minimum number Sum of money


vehicle vehicle required by Co-operativc set aside for project

Station wagon $60 000 4


$1 440000
Mini-bus $120 000 2

Let x be the number of station wagons and y the number of mini-buses.

(a) Write down two inequalities, one in r and the other in .v, which satisfy the given
conditions about the number of vehicles required. 121

(b) Write down an inequality in.r and y which satisfies the given conditions about the cost
of the vehicles and show that it reduces to r + 2y < 24. 12)
-
(c) The running cost of a station wagon is $900 per week while that of a mini-bus is $600. tr
The Co-operative does not wish to spend more than $12 000 per week on running costs.
N
Write down an inequality, in x and y, which satisfies this condition and show that it tr
reduces to 3x + 2y < 40. t2)
T
(d) The point (.r, y) represents.r station wagons and y mini-buses.

Using a scale of 2 cm to represent 2 station wagons on the horizontal axis and a scale of 2 cm to
represent 2 mini-buses on the vertical axis, draw the.r and y axes for the ranges 0 < x < 16 and
0 =.y -- 16.

Construct and indicate clearly, by shading the UITI}YANTED regions, the region in
which "r and y must lie.
[41

(e) A station wagon can make a profit of $250 per day while a mini-bus makes a profit of $600
per day.

Use your region to determine the number of station wagons and the number of
mini-buses the Co-operative should buy in order to maximise the profit. tzl

651€
6518

58

rl
The pattern given below is produced by joining the mid-points of the sides of each
equilateral triangle and then shading the equilateial triangle formed in the ceotre.
E()ns and
Sludy the pattern and the table that follows.

money

A
or proJect A

,W
) 000

B-C
Surting point Stage I Stage 2 Stage 3

Starting
cost point Stage I Stage 2 Stage 3 Stage 4 Stage 5

ilrm{rer of shaded )
i00.
mupnr,glg5 0 1 4" l3 n 121
costs.
llftumher of unshaded
It it - gles 1 3 9 I p r

I'6nxl I 4 l3 m q t

:of2cm (e)
x< 16a
Write down clearly the numerical values of l, m, fi, p,
Q, r and tr" t7I
(b) what fraction of the area of triangle ABC is not shaded at
in (i) stage 1,

(ii) srage 2,
(iii) srage 3?
fit of
tsI

:f
[2

s m/2/we(,

59
Canddate
Centre Number Nurnber

j
-'r .

-".\

,.[.,:d.:

2 hours 30 minutes

TO CANDIDATES
n,ame, Centre number and candidate number in the spaces at the top of this
page.
tnrr

answers in the spaces provided on the question paper'


b needed for any question it must be shown below that question.
cfr essentialworking will result in loss ol marks'

tables, slide rules and catculators may not be brought into the examination room.

TION FOR CANDIDATES


part question'
of marks is given in brackets [ ] at the end of each question or

FOH EXAMINER'S USE

Thls question PaPer consists pages a 1 blank Page.

tr7ao2
JI fTurn over
6l
NEITHER MATHEMATICAL TABLES NOR SLIDE RULES NOR CALCULATORS
MAY BE USED IN THIS PAPER

I (a) Calculate the exact value of A,072 + 0,9.

(b) Evaluate Oi *? giving your answer as an improper fraction.

(c) Express t as an exact decimal.

t1I

trI

trl

Express

(a) 17 30 as a time on the l2-hour clock,


(b) af nVs in km/tr.

fii
Ii
$il
i'
T:i

Atuwer (a) trl


(b) Izl

mN{Jlgltsv,

62
fq
Ex*.t't
UY

NE
roRs
(r) | - 4x2,
o) mn -7n - 5m + 35.

Altsllt'r (a) t1l

(b) t2)

The diagram shows an equilateral triangle


ABC with AB = 12 cm, f C = (i.v + l) cm and
tu .AC = (2r - 6) cm.
lrl Find the value of
IU
(a) r.

(b) .v.

(t, * t)

Answer (a) x = l1I

(b)y= lzl
trl
121

{IPI/,l(l291rS97 [Turn over

63
hr
Etaaincr's
t'sc

5 (a) Er.ress 1.7268correct to 3 signifiiant fiqures' I

(b) In 1994 the enrolment at a school rvas '500 pupils'


I

Giventhatthisenrolmenthaclincreasedby25o/ooverthattrflgg3.calculatethelt)t).1
enrolment.

Answer (a) trl


(b) l2l

The diagrams show diagonals of some special quadrilaterals.

(b) (c)

lti

ili

rll
Write down the name of the quadrilateral for each diagram'
lir

ii,

ll.

ii,,

ilir

lii trl
i,i

iri, tu
Ii
tu

fiatu{&ttl/sy,

64
(a) A man wished to save $250 in 11 months.

Giving vour answer correct to the nearest $5, calculate how much he should save each
month.
).]

(b) Calculate the simple interest earned on $600 invested in a bank for 18 months at a rate
o.f 24"/" Per annum.

rl

2l

Aruwer (a) $ tll


(b) $ t21

A metal box. whose mass is x kg, contains -y bolts and some nuts. There are twice as many
nuts as bolts.

(a) Write down, in terms of -y, the number of nuts in the box.

(b) Each bolt has a mass of 50 g and each nut has a mass of 20 g.
Find and simplify an expression for the total mass, in grams, of the box and its
contents.

I
Answer (a)
l
(b)

lftrult402Etllsyl lTurn over

65
Fq
Emim't
Usc

In the diagram APB and AQC are straight lines.

Given that PQ is parallel to BC, AP = 2 cm, PB = 5 cm and AQ = 3 cm' calculatc

(a) QC,

(b) the ratio of the'area of triangle APQ to the area of triangle PQB.

Answer (a) QC = ................................... cm

.. (b) LAPQ: LPQR = .'. ...' : ........

10 Given that log2 = y and tog-3 = n" express

(a) tn + n as a logarithm of a single number,

(b) log 9 in teirns of n,


(c) log f31 in terms of. m and n.

@t/lr{pm/sy,

66 .-'
FJ
Wt
ab

ll (a)

100o. Calculate R'S7"


In the diagram, PQisparallel to S7. fQn =48" and OfS =

(b)

State the order of rotational symmetry, about the point


O. of the figure above'

Attsr+'cr (a) RSl'=


ptY
/F,, I
Erumincr's I

lJsc I

I
12 (a)
I

The pie chart represents how a householcl allocated its January income on food.
school fees and other bills.

Given that the ratio p; Q: r =3:4;Z,find the value of q'

(b) The number of babies born in a hospital on the first seven days of JanuarY were

7, 3, 4, 11, 3, 8, 1.

Find (i) the mode,


(ii) the median of this distribution

Answer (a)q=- IU

(b) (i) Mo& = tu


(ii) Medin= trl

u^tMtusn

68

L
In the diagram, the points P,Q, R and Sare
consecutive vertices of a regular polygon
with fifteen sides.

Calculate

@I POR,
(b) Ros.

d.

A
Answer (a) POR= ................................... t2I

@ RasE ................................... tzl

l.t GiventhatA=(-?
i)""0r=(; :)
(a) calculate AB.
? 3 \

(b) find the inverse of A. ,* \ 'L 0

Answer fa) AB =
( )pt
(D)InverseorA= I ) pl

{rptiIE0lrs97 [Turn over

69
Velocity
(m/s)

Iime (s)

The diagram shows a velocity-time graph of a car during a period of 20 s. It starts from rest
andattains4velocityof,l?rnlsn4 s.Ittravelsat this velocityuntilatimeof lseconds. Then
the clr decelerates until it comes to rest.

(r) C;alculate the acceleration during the first 4 seconds.

(b) Given that the total distance the car travels is 180 m, find thc value of e

i, tu
13I

&ntlol,/Ttsy,
I

7A
Fot
Ewrrut\
Ur

12i:-

,t\\
^ A r' \

t2

ln the diagram, AB and CD are parallel chords of a circle centre O. t,

Given that AB = CD '= 12 cm- and OB = L0 cm, calculate the distance of. AB from the
centre of the circl-e. i'
I.

i:

Hence, or otherwise, write down the distance between the two chords. ii
ii
#

ili

I
I

I
i
!

f;

f;

/td&l/@UllS97 [Turn over

74
For
E oibw't
Us
l7 Given that m=4 x 105 and n= 5 xl02,expressinstandardfrl
(a) nt - n,

$, +.

Auswer (a) 12)

(b) ..........- 13I

lE O is the origin, and A and B are the Points (5, 12) and (1, a) respectiwly-
.-->
(a) Find AB in column vector form.
--->
(b) Given that BF = OA + zoi],find the coordinates of P'

Answer (a) AB =
(I I2l

l3l

.mnt{0a,n/Sn

72
19 (a) Simplifv 7.r i * -Sr- i . giving your answer with a positive index.
a-r -3
exPress x in terms of a'
(h) Given that
ffi= 2'

l2l

1unl(I2t l/s97 [Turn over

73
r Fot
r,ir''lt't
t
Uv
20 (a) Express 77 ro asa number in bc 5-

(b) a and D are digits such that ab,lnbase ten'b l0 r r+&'hcnrpL-{t = l0 x 4 + 8'

Express the two digit number bain the same form'

Hence, express in the same for -m and simplify

ab + ba.

l2l
tu
121

4mllt4o2tNsn

74
Fot
Etaltdls't
t2
2l A box contains 5 red sweets and 3 yellow sweets which are identical except for colour.

John takes a sweet from the box at random and eats it.

David then takes a sweet from the box at random.

Giving your anwers as fractions in their lowest tenns, calculate the probability that

(a) John takes a red sweet,

(b) both John and David take red sweets,


(c) both John and David take sweets of the same colour.

Answer (a)

(b)

(c)

{rfnr{Em/s9, [Turn over

75
F-
*l
tb j
(e) Cdculate the value of. xY'z given that t=1,y=-l 1=-L
4,01 x 5PB
(b) FindtheaPProximatevalue of givir3p-CDftmrest
\@
whole number.

(c) simpl,fy$ .*o-

NIIN&fIEilI

76

i
q

v
6

4
D o

L T
3

2 N
M
B G

(, \ F R

\\.
-l
\
-2

In the diagram O is the origin.


The diagram shows four squares OPQR, OABC, BCFG and LMNT, and parallelograrn
BCDE.

(a) OABC ismapped onto O PQR by an enlargement. Write down the coordinates of the
l:i
invariant point.

(b) LMNTis mapped onto BCDE by a shear.

State
(i) the shear factor,
(ii) the hvariant line.

(c) Describe fully the single transformation that maps OABC onto BGFC.

Answer (a)

(b)

(c)

t3l

{fr^t'l&E/Lts97 [Turn over

77

'ii
&
For For
Eramine -ruiaer
Use
Usc

The diagram shows the graPh of Y =


Jr '

(a) f)raw the tangent to the curve at the point (-1, 1)'

Hence, estimate the gradient of the curve at this point'

(b) Use the graph to solve the equationlxt * 1'

t3l

t3I

.frillt{]tttl!s91

78
For k For
$;*tr's Eramhlrr's
U* Lr Usc

25. (a) An aeroplane flying on a bearing of 029'changes its course by 34'.

Find the two possible bearings on which the aeroplane might then fly.

(b) North

The diagram shows a point O 8 km due north of another point P and the route of a
man who walks from P in the direction 05ff.

Using as much of the information given below as is necessary, calculate the distance
the man walks before he is
(i) as close to Q as possible,
(ii) due east of Q.

[tan 60' = L,732i cos 60o = 0,5; sin 6ff = 0,8660.]

12l

t2I

121

{mn/(t2a/us97

79
nME 2 hours 30 minutes

INSTRUCTIONS TO CANDIDATES
Write your name, Centre number and candidate number in the spaces provided on the answer paper/
answer booklel.
Answer all questions in Section A and any three questions from section B.
Write your answers on the separate answer paper provided.
lf you use more than one sheet of paper, fasten lhe sheets togelher.
Electronic calculators must not be used.
Allworking must be clearly shown. lt should be done on lhe same sheet as the rest of the answer.
Omission of essentialworking willresult in loss of marks"
lf the degree of accuracy is not specified in the questioh and if the answer is nol exact, the answer should
be given to three significant figures. Answers in degrees should be given to one decimal place.

INFORMATION FOR CANDIDATES


The number of marks is given in brackets [ ] at the end of each question or part question.
Mathematicaltables may be used to evaluate explicit numericalexpressions.

This question paper consists of i0 printeO pagps and?


MFK (0222) OF72003
o zsEc 1997
ffurn over

8l

,:
Section A [64 nrarksl

Answer all the questions in this scctirm-

(a) Solve the equation


3 - 2w = 5(rr'- 2). I2l

(b) (i) Factorise


(a) xz * 2x,
(b) x2-6x+8.
12l

(ii) Hence, or otherwise, find the L.C.M. of x2 - Lr and -r2 - 6r + & lrl
(iii) Express
ll + as a single fracrion in is loEtlG- t4l
fr ;I;;g

(a) Six exterior angles of an octagon are each .r" and the othe rs are (-r + frf d {6"-
Calculate x. t3l

(b)

In the diagram PS is a diameter of the circle passing through P, Q. R, S and 7'.


PSX and QRX are straight lines.

Given that RfS = 20" and QFs = 46", calculate


(i) Ris,
(ii) sfx,
(iii) rRQ.
14l

(c) Taking r to be 3,142, calculate the radius of a circle whose circumference is 33,5 cm. 12l

(d) Given that y varies inversely as r - 3 and ) = 3 when -r = 5, find the formula for y in
terms of x. t3l

'lrI(/rS97

82
I2I

'fhe Venn diaqram shows some of the results of a survev on {t0 Form One pupils who were asked
atrout thcir sporting activities.
t2l
E is the Set of pupils questioned in the survev.
ul // is the set of pupils rvho play hockey.
t4l S is the set of pupils who play soccer.
7- is the set of pupils.who play tennis.
The numbers of pupils in some of the subsets are shown.

(a) Calculate the nurnber of pupils who


t3l (i) play hockey,
(ii) play both hockey and tennis,
(iii) play tennis or both hockey and soqcer.
(iv) do not play any of the three activities.
l6l

(b) Use set notation to name the set with


(i) 6 pupils,
(ii) 14 pupils.
l3I

(a) Solve the simultaneous equations


3a - 2b =7,
5a+4b=8.
t3l
(b) (i) Find the gradient of the line joining the points A (3, -2) and B (2,?).
(ii) Hence find the equation of the line parallel to AB through the point (0, *l j).
(iii) Express your equation of the line in part (ii) in the form a-r + by = s.
t4I

(c) Solve the equation


t4l ,r2-3-r--5=0.
giving your answers to 2 decimal places" [4]
12l

t3]
[Turn over

83
In lhe diagram, ABCD is a quadrilateral.

Gi',en thar AB= 7 cm, BD =6cm, BC = C'D, BeO =76o an<J eEO --g-1ffrrc
(a) cED, ttt
(b) BC, t3l
(c) the area of triangle ABD, l2l
ldl AD, t4l
(e) the perpendicular distance of D from AB. t2l

Amwer the whole of this question on a sheet of plain paPer'


lines and arcs'
Use ruler and compasses only for all consructions and show clearly all the cortoruaion
(r) Construct on a single diagram
(i) a quadrilatenl WXYZ in which WX = l0 cm, w*v = 90o, XY =7 qrr, XfrZ = 120" an
WZ=8cm,
(ii) the locus of points which are 4 cm from W,
(ii) the locus of points which are equidistant from Y and Z'
ITII

O) Measure and write down the length of. YZ- trl


(c) The point R lies inside the quadrilateral such that WR < 4 cm and ZR
> RY'

Indicate clearly, by shading, the region in which the point R must lie.

E4
Section B [36 marksl

' rltrsx'er three rlttc.rrions in this seclitttt'

?,,, i\ tcacher has the exact amount to buv 66 notebooks at $2'2O each'
(i) LIos' nruch nloncv does she have altogether'l
will she bc ahle
1ii) If. instead. she clecicles to buy noteho6ks at $3.30 each' how many
ttl buv?
trl
the second andsubsequent
(b) 't'hc of a nerv hguse incrcases by 20"/o during.the.first year'In
'atue value at the lreginning of that year'
!"ars its value increases h1' 10"/o of iti
If at rhe beginning of 1990. the valuc of a new house was $60000, calculate the value of the
house
(i) at the beginning of 1991.

ttt (ii) at the beginning of 1993.


t.sl
{"
t3l {P'l t( \\'
t2l (c) A map is drawn to a scale of I : 4ffX)'
(i) the distance between them on the
l4l Two villages are 1.g km apart. calculate. in ientimetres,
map.
l2l by t.2 cm. Find the actual area' in square
(ii) A sports field is represented by a rectangle 3 cm
metres, of the sPorts field'
l4l

..r.! :'i
Lv Yi'l L* i\. .:I
r,'-
,I

i''
L
J

,l , "

// ' :*

18l

lu

' n,L

,)
lTurn over
,"r/,"'
ti
B5
r
'E ,j

The diagram represents an open rectangular water tank whose refs and base are made of concrete
30 cm ttiick. The external dimensions are 3,3 m by ?,Q m by 1.5 m

(a) Write down, in metres, the internal dimensions of the tank' tll
(b) Calculate
(i) the volume of water, in litres, when the tank is full'
(ii) the volume of concrete, in mr, used to build the tank,
(iii) tfre total external surface area, in m2, of the vertical walls of the tank. "t7l

(c) The vertical walls are to be painted on the outside. One litre of the paint covers -5 nr:. This paint
is sold in l.litre tins at $24,50 per tin.

Calculate
(i) the number of tins of paint that will be required'
(ii) the cost of the paint.
t4l

86
.\
I

Answer the whole df this question on a single sheet of greph PePcr.

'Iriangle A has vdrtices at (2. 2), (4. 4) and (0. 4). Triangle I has vertices at (-4, -2), (-tt. -2) and
(-6.-4).
(a) tlsing a scale of I cnr to represent I unit on each axis. draw axes for values of -r and y in
the ranges -10 <.r < l0 and - 6 <.v'< 14'
Draw and label the triangles A and 8. I2l
(b) Triangle I is the image of triangle A under a certain single transformation.
Describe fully this transformation. t3l

(c) I'riangle C is the image of triangle A under a reflection in the line y =.t - 4.

(i) Draw the line )' =.r - 4.

(ii) Draw and label the triangle C.


t3l

(d) Triangle D is the image of triangle A under a certain single transformation. Given that the
vertices of triangle D are at (-4,6). (0. l2) and (-8,'12),
(i) draw and label triangle D,
(ii) describe fully this transformation.

[Turn over

87
I

1l
l0 (a) List rhe inreger values of .r which satisfy all of the inequalities
l+.r<19-Lr<12
- and ll<2.r+3 <19.

(b)

(i) Use the diagram above to find


(s) the equation of the line /,
(b) the four inequalities which define the region R'
(ii) Giventhatp=5x+2y,findthemaximumvalueofpforintegralvaluesof
randyinR.

88
ll Answer the whole of this question on a sheet of graph paper.

The table below gives the cumulative frequency of the marks scored by 300 candidates in a
Mathematics examination.

Marks -r("/") "r<15 .r<30 x<40 x<50 x<60 x<75 x<80 r< lfi)
Number of 1 .. ..,tt

candidates who
scored this mark
0 30 72 145 206 28t 2n 300
or less

(a) If the pass mark was 5lolo. calculate the percentage, correct to 2 decimal places, of
candidates who passed.
I2l
(b) Copy and complete the frequency and frequency density table below,

Marks
x("/"\
15<r<30 30 <.r < 40 40<x<50 50 <.r < 60 60 <,r < 75 75<x<80 80<r<100
Number
of 30 42 19 6\ 7S ,'-1 10
candidates

Frequency 1>. \
2 4,2 5 ii6 0,5
density

[41

(c) Using a horizontal'scale of 2 cm to represent 10 marks and a vertical scale of 2 cm to


represent I unit, construct a histogram to represent these results.
l3l
(d) Use the table in (b) to calculate an estimate of the mean mark.
13l

&ilgn [Turn ovcr

89
r

t2

In the diagram, OP = 4e, OR =5b and RQ = 3a,.

(a) Express in terms of a and/or b


(i) oQ,
-->
(ii) RP,
--+
(iiil PQ.

OQ and RP meet at X _>


-__>
Given that RX = k RP, express RX in terms of a, b and k.
Hence show that Ok =5 (l - k)b + 4ka. t3I

(c) Given also that OX = O rQ:express OX in terms of a, b and &. tll


-> find the values of h and k.
(d) Using these two expressions f.or OX, 13I

(e) Find the numerical value of *" ruiuoTe. I2l

90
Gandidate
Centre Number Number

'.t.
.l
:..

TIME 2 hours 30 minules

INSTRUCTIONS TO CANDIDATES
page.
centre number and candidate number in the spaces at the top or this
write your name,
3I
Answer allquestions.
on the question paper.
write your answers in the spaces provided
it must be shown in the space below
that question'
It working is needed for any question
in loss ol marks'
3l Omission of essentialworking willresult the
tables, slide rules and electronic catculators may not be brought istts
Mathematical
ll examination room.
3I
INFORMATION FOR CANOIDATES
question or part question'
The number of marks is given in brackets I I at the end of each
1

FOR EXAfoIITdEM'S USE

ci 2 blank Pages'

MFK (0402) oK73371 ffiurn over


o zsEC'1997
9l
Fu
'aeiEr"
Utc

NEITHER MATHEMATICAL TABLES NOR SLIDE RULES NOR CALCULATORS


MAY BE USED IN THIS PAPER.

(e) Giving your answer as a common fraction in its haE Hrb*e of


(i) l*3,
(ii) I orH .

(b) Evaluate 0,35 - 0,162.

Answer (a)

(ii) .

(b) .............

2 (a) Round off.6734 correct to the nearesr 100.

(b) Express 39 000 in standard form.

(c) Write 0,02998 correct to three decimal places.

Answer (a)

(b)

7624 {0(ptrw97

92
Fa
fua*r\
Ut
_l
I
i
I
r!

In the diagram,z{Nis parallel to BM, OAN= 30P, eOA= 95o and OfrM = xo.

BO produced meets AN at C.

Calculate

(a) the value of xr


(b) NeB,

(c) reflex CiO.

Awwer (a'1x = I1l

@\NeB= tU

' (c) reflex Cio = tU

afrplrs/9 [Turn over

93
Fa
*l
IJ,
4 (c) ConvertasPeedof20metresPersecondintokilometresperhour.
respectively' by volume'
(b) chipo mixes lemon juice, sugarand water in the ratio 3 :2:7
35 ml of water'
to make lemon squash. tn rn'ating the squash chipo
used

Calculate the volume of sugar he used'

km/h tU

s (s) Solve the inequalitY -Z <3n +7 <27.


(b) write down the naturalnumbers which satisfy the inequality - 2 < 3n + 7 < 22'

A semi-circle has a radius of 18 cm, measured colrect to the nearest centimetre'

(e) Find the least possible value of the radius'


semi-circle'
(b) Taking n to be f , calculate the minimum area of the

{If,NT*T?
7621

94
''n
In the diagram, A, B,C and D are points on a circle centre O. '1*i j:.'- '
PAB and PDC are straight lines. 1
r.a
'
i
Given that eFD =23o, ADP = 75o and adn = 54o, calculate
I

(a) ABC, ;'-{.


u ..t

(b) P6D,
(c) AOD.

Answer @) eEC = tu

{If,rlr*y, [Turn over

95
r Fo
Emts't
lle
p varies directly as q and inversely as r.

(r) Find the equation for p in terms of q, r and a constant t-


(b) Calculate the value of &, given thatp - 2j when 4 = l(Ildr=il-
(,, Find the value of r when p = ll and I = 180.

Answer (a) P = lu

(b\k= tu

(c)r=.. tu

The shape shown above is an equilateral triangle.

(r) Draw, on the triangle, all the lines of symmitry.

(b) Write down the order of rotational symmetry about the ceotre of the triangle.

(c) Find the angle of rotational symmetry about the centre of thc triangle.

Answer (a) (on the diagram) [tl

76121 fifioun

96
Fo
E*\
tb

l0 It is given that f(x)=(x-3)2+5.


(a) Find.the value of (-2)'
(b)' Solve the equation f(x) = f(:2)'

tll

l2l

11 FactorisecomPletely

(a) Stt '20t,


(b) p2+sr- Ps-Px.

Answe,r

{I?trVY, lTurn over


7@4

97
For
F:onlnt\
Use

It is given that

8 - lx:r is an integer and? <.r < 5l.


A= lx:2<x<5)and
B is the set of odd numbers.

(a) List the elements of 8.

(b) Find
(i) A',
(ii) A w B.

Answer (a) IU

(i Given that log m = -6 and log n = S,evaluate

(a) logmn,
I
(b) logmt,,
I
{c) log , .

(b) trI

7624 10Uln$N

98
Solve the equations
Fot
Efrtr't
,tt
lI
i

(a) z-9=5;
r
I

(b) 7Y2 = 3Y.

ti

l2l

l2l

15 (r) Evaluate (e ,r(- ?)

(b)
"=(-! i)
(D Find the determinant of the matrix M'
(ii) Find the inverse of the matrix M'

(b) (i) determinant = """""""""" [1]

(ii) inverse is -.""""""" l2l

[Turn over

99
r
. Fot
Fnthct't
te
,
h
I
16 (a) Evaluate 11002 - 111r, giving your answer in base 2.
(b) (i) Express 11012 in base 5.

(ii) Evaluate 32, + 1101r, giving your answer in base 5.

(D) (i) IU

(ii) l2l

17 The probability that Tsitsi is late for breakfast is I whilst that for Tendai is |.

Giving each of your answers as a common fraction in its lowest terms, find the probability
that

(a) on a particular day,


(l) Tsitsi is early for breakfast,
(ii) they are both late for breakfast,
(iii) either Tsitsi or Tendai is late for breakfast,

(b) Tendai is early for breakfast on two consecutive days.

Answer (a) (i) tU

(ii) tll

7l&?.1 lufu!r9,

100
lt (a) Find the value of
(i) 7L *71\ *7,

(ii) (*)*
(b) Solve the equations
(i) 3' -- 9,

(ii) lb-3 =32.

'wru97 [fbnovcr
rol
.Fo For
F.*s't Emnbcr't I
t- Us Eoa
t
16 (a) Evaluate 11002 - 111r, giving your answer in base 2.
(b) (i) Express 11012 in base 5.

(ii) Evaluate 32r+ ll0lr, giving your answer in base 5.

(r) (i) tu

l2l

17 The probability that Tsitsi is late for breakfast is I whilst that for Tendai is |.
Giving each of your answers as a common fraction in its lowest terms, find the probability
that

(a) on a particular day,


(i) Tsitsi is early for breakfast,
(ii) they are both late for breakfast,
(iii) either Tsitsi or Tendai is late for breakfast,
(b) Tendai is early for breakfast on two consecutive days.

(ii) tu

(iii) I2l

tu

7@1 {rtnm
lm
(a) Find the value of

S*:,, (i) 7l ,71\ ,7,

(ii) (+#
(b) Solve the equations
(i) 3' = 9,
(ii) lb-3'=32.

Answer

{0lr!f,9, [T[tl ovct


rol
For
fuamia
Usc

On the grid, draw the lines

!=L, !=4, r+/=Q and Y=?'x'l'


Show,Lyshadingtheunwantedregions,theregionwhichsatisfiesalltheinequalities

Y>1, Y<4,' x+Y>'O and Y>Lr-l'

Answer (a) (on the grid) t3l

(b) (on the grid) tu

102
Fot
Etfr.
llr

In the diagram, nfC = W, AED = CED, AB = 5 cm, 8C = 25 cm and CD = 15 cm.


(r) Express sin Cfp as a common fraction in its lowest terms.

(b) Calculate
(i) BD,
(ii) the area of triangle ABD.

Answer (a) sin CBD = tu

t2l

l2l

{If,rlrs97 [Turn over

103
It, Fot
Erumimr
Fst "

Errrrto\
Ux
I t'tsc

UPPLY O
2l I

DOMESTIC CUSTOMERS

Fixed monthly charge $1s.80

EnergY charges Per unit:


first 300 units 16.30 cents
' allother units 3-5,00 cents

Below is an extract of Mrs Shumba's electricity bill


for the month of iVaY, 1997'

Present Number of 'Cost


Description Previous
reading reading units used Sc
Energy charge 96 689 97 039

Fixed monthly cilarge fffi 15 80

Sub-total

Development IrvY
(5olo of sub-total)

Calculate, for the month of MaY,

(a) the number of units of electricity used,


(b) the cost.of the electricity used,
(c) the DeveloPment Levy that Mrs Shumba will have to Pay'

7624

r04
Fot 5
Errtbc .;
lrll i
5+4o
22 It is given that q=6-3p'
d'
(a) Giving your answer as a common fraction in its lowest terms, calculate the value of q
when;r = -2.

(b) Express p in terms of g.

{ux,l/wv? [Turn over

105
In the diagram CBA and CDE are straight lines.

eiC = DEC = W, CE= 60 cm, CB = llcrn and BD = 8cnr.


(r) Name, in correct order, two triangles which are similar.

(b) Express tan treO asa common fraction.

(c) Calculate (il AE,.


(ii) the area of quadrilateral ABDE.

Answer (o) A and A

lrl

(c) (i) AE = .............,.........cm tU

(ii) area 3 .................... cm2 l2l

/mrl/*V,

106
___--

U The data shows the distribution of marks in Mathematics and Geography.

Mathematics: 22,20, 19, 18, 24, 19,23, 19,.?5.

Geography: 19, 23, 25, 2.1, 29, 1,2?, 24, 4"


(a) For the Mathematics marks, find

(i) the mode,


(ii) the median,
(iii) the mean.
(b) Calculate the mean for the Geography marks.
(c) In which distribution is the mean a better measure of central tendency?

Give a reason for your answer.

t1l

'IIP!f*Y, [Tun owr


tfft
of Fu
,aratt a?l EtorsF
Us Ust

25
,..!a

i::t/n

if;n

'i!.li:

::.;,i,

::,.*
iiiii
,i:ii
,lr.'
iii;
iil{

l,i
${+
!ill

','.:i;t

::.

TI

ti
\
al
The diagram shows triangles A, B and E. A
(a) 'On the diagram, draw and label
\r1

tf
(i) triangle C, the image of triangle A under a clockwise rotation of 90o about the E
origin,
A
(ii) triangle D, the image of triangle,4 under a shear of factor 2 with the x-axis as the c
invariant line. ll
b
O) Describe fully, for each of the following, a single transformation which maps
(i) ttiangle,4 onto triangle 8, I
(ii) triangle.4 onto triangle E. I
(c) Find the ratio of the area of triangle .d : the area of triangle E. I\

l1I
121

121

tzl

(c) area of triangle A: area of triangle E = tu

7d21 ry[fitwtn

108
q

TIME 2 hours 30 minutes

INSTRUCTIONS TO CANDIDATES
provided on the answer paper/
write your name, cenrre number and candidate number in the spaces
ans*et booklet.
from Section B'
Answer all questiOns in Section A and any three questions
provided'
write your answers and working on the separate answer paper
lf you use more than one sheet ol paper, fasten the sheets together' l

Electronlc catculators must not be used'


the rest of the answer'
Allworking must be clearly shown. lt should be done on the same sheet as
omission of essentialwor-king willresult in loss of marks.
ll the degree ol accuracy is not specified in the question and if the
answer is not exact, the answer should
be given to three signilicant figures.

NFORMANON FOR CANDIDATES


question or part question'
The number of marks is given in brackets I lat the end of each
Mathematicattables may be used to evaluate explicit numerical expressions'

7652
MFK (0403) QK73372
o zsEc 1997
Furn over

r09

.,,,
Section A [64 marks]

Answer all the questions in this section.

1 (r) Simplify 6x-y+32-(4x-2y+52). l2l


(b) Factorise (D q2 -3q - to, lzl
(ii) (5 - p)z + e(5 -p). l3l

(c) Simplify
32
(2r+l)(3-x) x-3 l3I

2 (e) Solve the equation l(r - t) =16 -zy. l2l


(b) Giventhaty =l-?:andz =l+b,express y2+4yzintermsof .r.Simplifyyouranswer' I3l

(c) The simple interest on $20 fr)O invested for 9 months was $2175. Find the rate of interest P€r
annum. I3l

3 (r) (Note that on a velocity-time graph the point (3, 20) denotes that after 3 seconds the velocity is
20 metres per second.)

The velocity-time graph of a bus moving with uniform acceleration Passes through the points
(3,20) and (5, 35). For this journey calculate
(i) the acceleration, l2l
(ii) the time when the velocity is 30 metres per second, 121

(iit) the distance travelled during the time interval from t = 3 to, = 5. l2l
(b) The plan of a house is made using a scale of 1:150.

C;alculate
(i) the length of a line on the plan which represents a side of 12 metres on the house. l2l
(ii) the volume, in cubic metres, of a pillar whose dimensions on the plan give a volume of
20 cubiccentimetres. t3l

7632 7(

ll0
a (a)

(i) In the diagram A, B and C are points on the circumference of a circle centre O.
EIis the tangent to the circle at the point B and AOCT is a straight line.

Given that CFI= 38" calculate

(a) i)es, tu
(b) BAc, 121

(c) stc. l2l


(ii1 D is a point on the diagram, on the same side of BC as O, such that nDC = 52o.

Does the point D lie on the circumference of the circle?

Give a reason for your answer.

(b)

In the diagram, ABCD is a kite in which AB = 4 cm and AD = 5 cm.

Given *at BAD = BeD = x)o, find


(i) the length ot BD, t3l
(ii) the radius of the circle which passes through the points A, B, C and D. I2l

[Turn over

lil
(a)
A

336 B

60 4t3 c
206 401 155 D

123 2N 169 243 E

2t7 224 216 205 78 F

The table shows the distances in kilometres between the to$,ns A, B, c, D, E and F.

(For.example, the distance from C to D is 155 km and the


dlsrerce from D to A is 206 km, so
the distance from C to D to .4 is 155 + 206 = 361 km).

Two of the routes from.4 to D are as follows:


Route L4 to F to E to D,
RouteZ AtoBtoEtoD.
Calculate
(i) the distance of D from A on each of these routes,
t4l
(ii) the difference between the shorter route and the distance between .4 and D given in the
table.
I2l
(b) The premium of a certain insurance policy is calculated using the formuta

g =(M - r): rcw


R
where $Q is the sum assured,

$M is the monthly premium,


$F is the policy fee
and R is a factor obtained from the insurance tables.

Civen that Q = 50 000, F = 20and R = 45, find the value of M.


I2l
(c) The price P of a commodity varies partly as the demand D and partly as the availability .4.
(i) Write down the expression for P in terms of D and z{ using the constants c and k.
tll
(ii) Given that P= 19when D = 5 and A= 2 and that P= l7 when D 3and A= 4,write down
=
two simultaneous equations and hence find the value of c and the value of k.
I4l

7652 .(n//2/vty,

lt2

l.
Answer the whole of this question on a sheet of plain PaPer.

lJse ntler and compasses only. All constntction lines and arcs must be clearly shown.

(a) On a single diagram construct


(i) triangle ABC inwhich AB = AC = 9.5 cm and BiC = 6Oo' t2]

(ii) the locus of points which are equidistant from AB and AC, 121

(iii) rectangle BXYC,on the same side of. BC as A. which is equal in area to triangle ABC. t3l

(b) (i) Measure and write down the length of the perpendicular from A to BC. [l ]

(ii) XY represents a certain [oc[s D"t.tiU. this locus fully' l2l

Section B [36 marks]


6 km, so

Answer three questions in this section.


ic
Each question in this section carries 12 marks.

(a) l)' "r


Solve the equation
i(
3r:=5r+l
giving your answers correct to 2 significant figures. t5l
t4l
in the (b)
12i.

In the diagram, ADQ is a straight line, AD = 5 cm, DC = 4 cm, BD =3 cm and the area of
I2l triangle ebo is 4 cm2.
,4.
lrl (i) tbB, l3l
l down (ii) BC. [4]
t4l

,rII{/2tW97 [Turn over

ll3
Take n to Ue f.

In the diagram, the shaded area represents the uniform cross-section of a concrete block which can
be used as a water run-way.
XFS =PfR = SRq = nOy =9f, RS= 4Ocm, PS = QR= 20cm and PX = Ye = 6cm.

The cross-section of the water channel XZY is a semi-circle and the block is 2fi) cm long.

(e) Calculate
(i) the area of the cross-section of the block (shaded region on the diagram), l3l
(ii) the volume of concrete in the block, in cubic metres, I2l
(iii) the mass of the block, given that the density of the concrete is 22ffi kd.'. t2;
(iv) the surface area of the water channel in square centimerres. t2l
(b) The ends of the block are closed to form a water trough. Calcutate the volume, in litres, of
water in the trough when it is full. I3l

n4
Answer the whole of thb guestion on r single sheet of grrph p.per.

The tahlc below shows some of the values of the function v =.r't.

1
,l -1.5 -t 0 I 1,5 2

t' p -3.4 -l 0 I 3,4 tt

(a) Find the value of p. Ir]


(b) Using a scale of 2 cm ,o rcpresent 0,5 units on the x-axis and 2 cm to represent 2 units on the
y-axis, draw the graph of v =.r' for the range -2 < x < 2.
I4l
(c) Showing your pethod clearly. use your graph to estimate the value of {Ea'. t2I
(d) Using the same axes and scates, draw the graph of y = 3x - l. l2l
(e) Use your graphs to estimate the roots of the equation x3 - 3x + I = 0. l3l

r0 Giventhat#=(I).&=( andoistheorigin,
_i )
(a) find
(i) the bearing of O from P, l3I
(iD I p? l. t3I
(b) R is a point on the line OR such that Qi = k OF.
iil Find the equation of the line Qfi in the form y = mx + c.
[.31
(ii) Write down the coordinates of point I
where the line Qfi meets the y-axis.
[3] Srate tfie trunsletioi v€ct6r r{/[io[ maps 0 6ilto fl.
ttl
t2l
[2,
t2l
{ji

ITurn over

l15
f
1l (a) A school hall, with 50 rows of seats, has 10 seats in the first row and one more seat in each
successive row.

When students write an examination they sit in such a way that thev leave a vacant seat
between any two of them in any given row. The table below shows the number of seals and the
maximum number of students in a given row.

Row Number of seats Number of students

I l0 5

2 l1 a

3 t2 6

4 l3 7

5 t4 b

l8 c t4

d ,47 e

50

Study the table above and use it to find the value of


(i) a,

(ii) b,
(iii) c,

(iv) d,
(v) e.

t5l

7652 ltfraw\n
76

il6
(b)

40 25 r5

A dart (small arrow) is thrown at the square target of side 40 cm shown above. There is a
shaded region inside the targeg the dimensions of which are 25 cm square on the outside and
15 cm square on the'inside. Assuming that the dart hits the target in a random manner, find the
probability that it will hit the shaded region. t4I
(c) In a class of ,00 pupils, 24 study mathematics, 18 study geography and 11 study neither of these
subjects.
A pupil is chosen at random.

Find the probability that the pupil studies geography but not mathematics.
t3l

[Trr ctr
tt7
i

l2 (r) Mrs Shoko has 300 cows' lM goats and 76 pigs'

(t) construct an accurate pie chart representing the above information. I4l
(ii) Two animals are selected at random'
calculate the probability that one is a goat and the other is a cow. lrave your answer as a
fraction in its iowest terms. t3l

Age (years)

The diagram illustrates the distribution of the ages of the pupils at a certain school.
(i) State the name of this type of diagram. tu
(ii) Calculate the number of pupils in the 16 to 19 age grouP' I2l
(iii) In which of the age ranges does
(a) the modal value occur,
(b) the median value occur?
{21

7652

ll8
Canclklate
Cantre Number Number

candidate Name

ZIMBABWE SCHOOL EXAMINATIONS COUNCIL


in collaboratlon wlth
UNIVERSITY OF CAi'BRIDGE LOCAL EXAUINATIOilS SYNDICATE
General Certllicate of Educatlon Ordlnary Level

MATHEMATICS 4009/1,4a29t1
PAPER 1

Thursday 28 irAY 1998 ,Attemoon 2 hours 30 minutes

Cardiclates ans:wer on the question paper.


No additional malerials are required.

IHE 2 hours 30 minutes

}(}TRUCTIONS TO CANDIDATES
Wiite your name, Centre number and candidate number in the spasos at the top of this page.
Arpwer all questions.
Write your answers in the spaces provided on the queslion paper.
working is needed for any question it must be shown below that question"
i{

Cmission of essentialworking wilt result in loss of marks.


Iathematlcal tables, sllde rules and calculators may not be brought lnto the sxamlnailon rloom.

]SORIIATION FOR CANDIDATES


The number of marks is given in brackets [ ] at the end of each question or part question.

FON EXAilINER'S UsE

Thls qu$tlon paper conslsts of 20 p,rlnlod pegos.


:---
5K (0.{9) OF86Os0r2
c zsEc t{*r8
fTurn over

ll9
NEITHER MATHEMATICAL TABLES NOR SLIDE RULES NOR CALCULATORS
MAY BE USED IN THIS PAPER

(e) Express
(i) 0,76 as a fraction in its lowest terms,
(ii) 160 x 10-r in standard form.
(b) Find the exact value otz! x 2). - -

Aruwer (a) (i) l1l

tu
(b) IU

2 (r) Giving your answer in decimal form, evaluate


(i) 5,c2-25,
(ll) 452 x 0,(ts.
O) Find the exact value of 780 + 0,013.

{Irlrlr.t 2lvl698

t20
'-'J
u*l
Fo.
E*l

3 Solve the simultaneous equations


I

3x+4y-5,
I
5x+21=4y.
I

t3l

In the diagram, ABCDis a kite in which AB =7 cm,BC =24cmafiefrC = 90'


(a) Write down, as a comnon fraction, the value of tan
pAC'

Calculate the area of the kite.


9
I

tu
i,t
t2l
It
li:'l
llt tr

l"ll
{In tr{nf,,trs9E [Turn ovcr
h{

l2l
IoJh" diagram, points A, B, C a$ O are on level ground and 0 is due south of .A.
AOB = f, BOC - (Lr + 10)o and AOC = bo.

C,alculate

(r) the value of x,

(b) the bearing of


(i) O from B,
(il) Ofrom C.

Answer (a) x = tll


.
(D) (i) tu
(ii) tu

{tn tr{E[fl/slt

122
lt

6 ExPress as a single fraction


3
x-2' 5x+l

7 Temperature "C: 12, 14' 16' 12'23'17'12'14'


recorded in a town on some days'
The data show the maximum temPeratures

(a) Find the median of the distribution'


(b) On another^- day a .^6^a?qr
r^-. ^ temPerature of x
oC was rtlcorded. Given that the mean
oc, calculate the value of x.
,"*p"r",ur" toi'.[ these'days was 16

il

IU

tzl

{rll,l,lrEi,lrsgll lTuro over

123
?d,
For
rilrrl Etomircr's For
le Usc
Etamia
Usc
8

In the diagram, P, Q, R and S lie on a circle centre O and SIis a tangent.

Chords SPand SR are equal and PSf = 4ff.

C.alculate
) '
(e) POs,
(b) PQS,

(c) spR.

{IIUtr{ngtrs9t

t24
In this question take rr ,oA"2]'

The diagram represents the plan of a church. The floor consists of a rectangle and a
semi-circle. Alldimensions are in metres.

Calculate

(a) the diameter of the semi-circle.

(b) the perimeter of the floor.

tu
tzl

{rlvl/{t2lvt/sgll [Turn over

125
Fu For
Fo Ilamincr't Eruminq':
l-*,t's Use Usc
U*
l0 From the top of a vertical cliff 56 metres high, the angle of depression of a ship anchored on
the sea is 3j". Using as much of the information given below as necessary. calculatc the
distance of the ship from the foot of the cliff'

[sin 35'= 0,57; cos 35o = 0.82: tan 35o =


0,70.]

Answer m t3l

ll (e) The distance, rounded off to the nearest l0 km. between Masvingo and Bcitbridgc
is 290 km.

Write down, correct to the nearest kilometre, the minimum distance from which -this
approximation could have been made.

(b) A household is allowed 450litres of water a day.

Calculate, in m3, the amount of water the household is allowed to use in the month of
APril.

Answer (a) knr tll


m' l}l
{rll,t/4asl/sefi

t26
t2 Given that

szr
= 4s2 -7,
express s in terms of ,.

Answer s=

13 (e) Express f; u pet."ntag,e.


"t
(b) Write down the value of

2xl}z +4x l0+7 x l0-r


(c) Given that 9p = 7q, find the ratio p : q.

{IUU'NUI'$II

127
I ttr
F- titnntr't
F*r\ {,'rr
tt
14 (a) lf. X = m -3nand y =2n -nr, find 3X + Y in terms of nl and rl.

(b) Solve the equation

3p2+8p+4=0.

Aruwer (a)

l5 Given that x is an integer and that - 13 <.r € 35,

(e) write down


(i) the least value of x,
(ii) the greatest value of x, such that x is a perfect cube;

(b) find two positive values of x which have a remainder of I when divided by both 3 and 5.

{If,rl/'02f,r1/S9ll

128
ThediagramispartofaregulardecagonABCDEFGH/Jwithcentre0.

(a) State the number of lines of syrgnetry of the decagon'


\
(b) Calculate
(i) the size of the interior angle of the decagon'
(ii) c6r.

{Urr-ll{[wl/S9l [Turn over I

129
For For
Elolrr,imr's Examincr't
Usc Uv
l7 (a) A train left a station at 14 26.and arrived at its destination after travelling for 3l hours.
Find the time, on the 12-hour clock, at which the train arrived at its destination.

(b) A girl takes 8 minutes 20seconds to walk to school. She can run 2j times faster than she
can walk. Calculate the time. in minutes and seconds. she would take to run to school.

tugtrtrEfllrsr

130
Fot
F.rr,niaer't
Ust

t0
Iime (0 in seconds

The diagram is the velocity-time graph of a car that decelerated uniformly from a velocity
of 30 m/s until it came to rest after l0 seconds.

Calcutate

(a) the retardation of the car,

(b) the distance travelled from t = 2 to r = 5.

tu
t3l

4q[Ari(r28rrrs98 [Turn over

t3t
tu For
F.rnhrilr Err,mbrr's
U* Use

19 (a) Solve the equation


3d _4=z\.
43'
(b) Given that f(x) = 5x2 - kx + 6and f(- 3; = 15, find the value of k.

(b)k= l2l

0nrcmrtrsn

132
20

/t\
t\I
/-*--------

The diagram represents two. similar square pyramids A and B with base edges 5 cm and
l5 cm respectively.

(e) Write down, in its simplest form, the ratio


(i) the height of. A: the height of B,
(ii) the volume of. A: the volume of B.
(b) Given that t'he area of one triangular face of pyramid A is 20 cm
2,
find the total area of
the triangular faces of pyramid B.

l1l

tu
I2l

'lllf,rV{rllClrs9t [Turn over

133
For
Exuniacr't
Usc

2l (a) Express 44, as a number in base 2'

(b) Find x such that xr+ 44r= 403s.

n Given that log 3 =0,477 and log 5 = 0,699, find

(a) log 1.5,

(b) tog225,

(c) log 0,6.

tu
t}l
l2l

{xn t/{}zurlsqt

134
F
.€D

The diagram shows LPQR and LP2Q2Rz'

(a) Mark and label point ,S such that RT= (


-: )
reflection in the line y = 1'
(b) Draw and label A P,Q,R , the image of a PQrR under a
aPQR onto ;\P2C7ft:'
(c) Describe completely the single transformation that maps

Answer (a) on diagram tri


(D) on diagram t?l

(c)'..-"..."....

l2]

{rwl/{,1ll,1/sgn [Turn over

t35
For
Fct fuoniurt
;,rl;etrrl Usc
Usc
U (a, Study the pattern below.

line 1 20 20

line 2 ,$0 60

line 3 P 120

line 4 80 q

line 5 100 300

line 6 r
lrll
tlil
llll
rttl

linet I I 720

Use the pattern to find the value of


(i) p,
(ii) g,
(iii) r,
(iv) t.

(b) A straight stretch of a farm bourldary 2,4 kilometres long is to be fenced.


Fencing posts are put at 5 metre intervals, starting at one end.
Calculate the number of Posts used.

Answer (a) (i) p = .. tll


(ii) q = t1l

(iii) r = .. t1l
(iv) t = .. tll
l2l

awt/rorE,t/s9E

r36
_:{
Fw
Errla*r\
U*

25 The following information is displayed in a wholesale shop.

Number of chic(ens Price per kilogram

1-10 $18,50

ll -20 $17,50

21 or more $17,00

Given that. in each case, the average mass of the chickens is 1,5 kg, calculate

(a) the cost of 50 chickens

(b) the cost of chickens of total mass 12 kg,

(c) the number of whole chickens that would cost $357,50'

QUESTION 26IS PRINTED ON THE NEXT PAGE.

.iln l/{02rvl/s9t [Turn over

t37
for
.,*,1\ Exambtr't
U* Usc

xt

A circle with centre o, where O is the origin, meets the axes at A, B and C as shown.

A is the point (7,0).

(a) Write down


(i) the radius of the circle,
(ii) the coordinates of B.

(b)
! , calcutate the area
Taking rr lo be of the shaded region AOB.

(c) Find
(i) the gradient of the straight line BC,
(ii) the equ4tidn of the straight line through .4 parallel to BC.

l(xplrao2&rrs98

138
Fot
Eramim
Usc

ZIMBABWE SCHOOL EXAMINATIONS COUNCIL


in coltaboration with
UNIVERSITY OF CAMBRIDGE LOCAL EXAMINATIONS SYNDICATE
General Certificate of Education Ordinary Level

MATHEMATICS 402812
PAPER 2
Wednesday 3 JUNE 1998 Afternoon 2 hours 30 minutes

Additional malerials:
Answer PaPer
Geomelrical instrumenls
Graph paper (3 sheets)
Malhematical tables
Plain paper (1 sheet)

TIME 2 hours 30 minutes

INSTRUCTIONS TO CANDIDATES
Write your name, Centre number and candidale number in the spaces provided on the answer paperl
answer booklet.
Answer all questions in Section A and any three questions from Section B.
Write your answers on the separate answer paper provided.
lf you use more than one sheet of paper, fasten the sheets together.
All working must be clearly shown. lt should be done on the same sheet as the rest of the answer.
Omission of essential working will result in loss of marks.
lf the degree of accuracy is not specified in tho question and if the answer is not exact, the answer
should be given to three significant figures. Answers in degrees should be given to one decimalplace.

INFORMATION FOR CANDIDATES


The nurnber of marks is given in brackets [ ] at the end of each question or part guestion.
Mathematicaltables or electronic calculators may be used to evaluate explicat numericalexpressions.

This que-tion paper consists of 11 printed pages and 1 blank page.


SB (SM) QF86065+
o zsEc 1998 ffurn over

139
Section A [64 marks]

Aruwer a,ll the quations in this section.

f (e) Given thata =3, =-2.b


and c=-S,find the value of
(i) ?t +7b - c,
(ii) b'-c"
(iii)
t-*
(b) Factorisecompletely
(0 Z8x'y +7xy,
(ii) 5 - 45cz

,r),
2 (i )*o'= i
Given,fr., a, = :1 (

find

(e) B-3A, t3l

(b) AB, 121


4

(c) thI Oeterminant of A, l2l


(d) the matrix c such that AC = (; i )
121

lr()
"-

/!
v)

\a(,^

t ."7

.\/'
l.

x
U
In the diagram, WXYZ is a trapezium in which WZis parallel to )(y arndWXY = ff.
Given thatW/Z= 5 cm, XY =9 cm and VfrX =57, calculate

(e) WX, l2l


(b) wY, l3l
\-.
(c) wfz, bY t3I

(d) the area of triangle WYZ. t3l

I
I
t
Three" villages
BAC = l25".
A, B and C are joined by straight roads with AB = 9 km, AC = 6,5 km and

B
Calculate
rt
(a) the time it would take to walk from A to B at an average speed of 1,8 m/s, giving your answer in
hours and minutes to the nearest minute, ' I3l
t
(b) the length of the road BC. ttl
The road from C to A continues in a straight line to another village D. Angle npp =7f .
(c) Calculate the length of therroad AD. lsl

[Tura over
l4l I

I
(a)

In the diagram, LMQ is an isosceles triangle with QL = QM and LQM = 50". The straightline LMN
YNis parallel to QM and PY is parallel to QL.
is parallel to QP,

Calculate
(i) Qiu,
(ii) uap,
(iii) p?u.
t4l

(b) .,

lnthediagram, DEFG isacyclicquadrilateralandEFisproducedto H.DFisadiameterof the


circle and intersects GE at X.

Given that EdF= 35" and rfc =48o, calculate ,


(i) GFn,
(ii) DFE,
(iii) ofrr.
t6l
(a) Write down, in set notation, the sets represented by the shaded regions in the fonou,ing
venn
diagrams.
tsl
(i)

(ii)
z
v

(iii)
e

(b) Answer this part-of the question on a


lingte sheet of plain paper. use ruler and compasses only.
All construction lines must be clearly shown.

Construct in a single diagram.


(i) LXYZ with sides ,YY = 8,5 cm, XZ = 6,8 cm and yZ = 10 cm.
(ii) the locus of points which are 4 cm frorn X
(iii) the locus of points which are equidistant from x and Z. _*

15l

!iv9n that P is a point su_ch that E = lP: P lies inside the LXyZ construcred above ),
T = {P : PX <4 cm} and S = {p : pX > Zpl,
indicate, by shading, the region In S.
tzl

ffurn over

143
Section B [36 marks]

Answer lltee questions in this section.

I ,l (e) John invests $12 000 in a bank. The bank pays2l,So/osimple interest per annum.

Find how much the money invested will amount to after 2f years. t3l
o) a second-hand car is offered for sale at a cash price of $lg 900.
(D Calculate the cost price of the car if the cash price includ es11o/o sales tax.
The car can alrc be bought on hire purchase by paying a deposit of 33f o/o
of the cash price.
(ii) Calculate the amount of the deposit.
The remainder is payable over three years after a finance charge on the balance
has been
added. The-finance.charge for the tluee-year paFnent perioi is calculated
once at the
beginning of this period ana is at the rate of.3|yo per annurn.
(iii) Calculate the amount of the fuiance charge.
(iv) The total amount owing is then paid in equal monthly instalments.
Calculate the amount of each instalment.

tel

IM
\I

E (a) Solve the inequalitY

-21<8r-5<75
and illustrate the solution set on a
number line' t3l

his CI-hectare farm'


(b) A farmer Srows tomatoes and beans on
to plant a hectare of beans' He has only
It costs $80 to Plant a hectare of tomatoes and $50
$4000 to meet the exPenses'

Ittakes24hourstoplantahectareoftomatoesand16hourstoPlantahectareofbeans.The
farmer has at most 1224 hours for planttng'

Taking , to represent the number of.hectares:1,*t^:"il:t",t-::t,'=t:::lfit3f:T;;ff,tr;j


than r > 0 and b -- o, which satisfv
I*T3lL'":t"'fi:::s, write down tr,r"" ii"qr"riiiur,otrr"r t3l
t3l
the above conditions'

single sheet of gaPh PtPer'


(c) Answer rhis part of the question on a

Thepointswithcoordinates(x,y)satisfythefollowinginequalities:
y>0,
x+y<10,
5x +2y > 20,

2y<x.
(i) Tii:-:l :":}fl: :::'"t:*:i:Ti[Y;::5;l[?f;i
Using a scale of 2 cm to rePr:sel:2 points'(x,v)
*Tffi:ff":L:,T#i;ffiil; Ii**t"a regions, the region in which the f4l
t4l
must lie.
value of x + 3y' l2l
(ii) Use the graph to estimate the maximum

[Turn over

145
In this question take r ,oh"f;.

In the diagram, BOC is a sector of a circle centre O and radius 6 cm. BOC =L4V, AOB = 9ff and
AO -- 9,5 cm.

(a) Show that AB = lL,Z cm, correct to 0,1 cm. trl


(b) Calculate
(i) the length of the arc BC, l2l
(ii) the perimeter of the figure ABCOA, I2l
(iii) the area of the figure ABCOA" t3I
(c) If this figure is the cross-section of a solid of length L5 cm,
calculate
(i) the volume of the solid, I2l
, (ii) the crfived surface area of the solid. l2l

146
l0 Answer the whole of this question on a single sheel of SfaPh PaPer. !

The following is an incomplete table of values for the graph of y = (x + 2)(5 - x).

x -2 -1 0 I 2 J 4 5

v 0 tn 10 t2 n 10 6 0

(a) Calculate the values of m and n. t2l

(b) Taking 2 cm to represent I unit on the.r-axis and2 cm to represent 2 units on the y-axis, draw
the graph of. y = (.r + 2)(5 - .r) for -2 < .r < 5. t4l

(c) Write down the maximum value of y from the graph. tll
(d) Use the graph to estimate, correct to one decimal place, the solutions of the equation

(x + 2)(5 -r) = 3. tll


nd (e) On the same axes, draw the graph of a suitable straight line and use your graphs to estimate the
solutions of the equation

trl (.r+2)(5-x)=x+6. l2l


(f) By drawing a tangent, find the gradient of the graph y = (x + 2)(5 -.r) at the point x = 2. 121

l2l
Izl
t3l

t21

[Turn over

147
1l Answer the whole of this question on a sheet of graph paper.

Table A, below, shows the cumulative frequencies and Table B shows the frequencies
of the masses
of 80 calves.

Table A

Table B

mass (kg) fl)<ra.60 l6O<m<70 70<m<80 80<rn<90 90<ra<lfi) 100<rz<110 110<rn<120


&equcncy I 5 U v 21 w I

(r) Calculate the values ol U,V andW.


I2l
o) State the modal class.
tu
(c) Using a horizontal scale of,2cn to represent 10kg and a vertical scale of
2cm to represent
10 calves, draw a smooth cumulative frequency curve for this distribution for
50 < tn < 120. t4I
(d) Estimate, using your graph, the number of calves whose mass is more than 25 kg.
I2l
(e) Two cdves are selected at random.

Calculate- the probability that one calf has a mass more tha.n 80 kg but less than or equal to
90 kg and the other has a mass more than 100 kg.
t3l

148
(a) ltOA-p+5q, Ob = 7p+ 3q and AE = 3hp + (rr - t) q, tud rb v.bdf dL
(b)

In the diagram, LPQR is t'he image of LPBA under an enlargement with centre P and scal
factor ), Pe = 8 cm and AB = 5$ cm.
(i) Calculate
(a) the length of AR,

(b) L PAB .
the ratio
A PRQ.
15. l

(ii) LPCB is the image of. LQRB under an enlargement.


For this enlargement, find
(a) the centre,
(b) the scale factor.
t3

149
----=
Candidate
Centre Number Number

Candidate Name

ZIMBABWE SCHOOL EXAMINATIONS COUNCIL


in collaboration with
UNIVERSITY OF CAMBRIDGE LOCAL EXAMINATIONS SYNDICATE
General Certificate ol Education Ordinary Level
MATHEMATICS 4008/1 ,4A28t1
PAPER 1

wednesday 11 NOVEMBER 1998 2 hours 30 minutes

Candidates answer on lhe questron paper.


No addilional materials are required.

TIME 2 hours 30 minutes

INSTRUCTIONS TO CANDIDATES
Write your name, Centre number and candidate number in the spaces at the top of this page.
Answer allquestions.
Write your answers in the spaces provided on the question paper.
lf working is needed for any question it must be shown in the space belgw that question.
Omission of essentialworking willresult in loss of marks.
);
Mathematical tables, slide rules and electronic calculators may not be brought into the
examination room.

INFORMATION FOR CANDTDATES


The number of marks is given in brackets [ ] at the enJot each question or part question.

This question paper consists of 17 printed pages and 3 bhnk pages.


MFK (0653) QK87688
fzsEc 1998 l5l fTurn over

/
For lir Ft
Etamiacr't trgnrim'r I,lamt
(.'rr Ut
Urt

NETTHER MATHEMATICAL TABLES NOR SLTDE RULES NOR CALCULATORS


MAY BE USED IN THTS PAPER.

I (a) Giving your answer as a common fraction in its lowest terms, find the value of
(i) 1_1 ,
(ii),1*9.
(b) Evaluate, giving your answer as a decimal.
0.57 + 1.469.

Answer (a) (i) tll

2 (sl Write 7,316 x 10-r as a decimal.

(b) Express 0,058 96 correct to three significant figures.

(c) Express f; as apercentage. i

{trul/w98

152
\ $\ $rracer\d\\(u.lr\eN\\\\Nst\\e-!K\-\e$$tQ.*\ttssst-\rrltrrgts\\ts-1'Cx\\\et\e
maximum temperature was l3 "C.
Calculate the temperature range on that day.

(b) An empty swimming pool is in the shape of a horizqntal rectangular trough.


It is 24 m long and l0 m wide. Rain fell into the pool to a depth of 4 cm.
Calculate. in cubic metres, the volume of water which fell into the pool.

Answer (a) trl

Given that r = 3., = - 5 and u = 7, find the value of

(a) rt2.

(b) u-2(r+t).

\9)e -
^)-')

,tll(rllwt (ll
lTurn ovei

153
,l For Ft
turi,,at's Etumint's Exomt
Uv v Utc Us

5 Giventhat4<r < 6and -2< y <T,calculate


(a) the greatest possible value of .r - y,
(b) the least possible value of x2 + y2.

6 Solve the simultaneous equations

I=1-3x.
2x+Y=5'

Answcr x =

{Drtr*x

154
In the diasram. ABC, AFD and BFE atestraight lines, BE is parallel to CD, BAF =8T.
eEF = 56iand gfi6 = 75'.

(a) Write down the size of ee D.

(b) Calculate the size ot EDF.

Given that/varies inverselY as w,

(a) write down an expression for/in terms of w and a constant ft.


/
Given also that f = 15 when w = 35,

(b) calculate (i) the value of the constant &,

(ii) w when/= lQ,J'

(bxi) k = ll l

(ii) lrl

4lrrvl/$r8 [Turn over

155
ft'r fit
F.xoatit
trn?t'r I [-tumincr't
/sc I Uv U*

In the diagram, A, B, C and D are points on the circle, centre O.


AB is a diameter of the circle and AD is parallel ro OC.

Given that e6O = 70", calculate

(a) BOC,

(b) B60,
(c) aeo.

(D) IU

156
l0 (a) Evaluate 432r+ 414r. giving your answer in base five.

(b) Express 2t + 23 + 2 as a number in base two.

(c) Convert 21,,, to a number in base two.

ll Given that log,,,7 = 0.845.

(r) log,o49,

(b) log,r70,

(c) log,rY.

tu

IU

(c) lrl
{ftr*r lTurn ovcr
i

157

\
Era
i

12 On a map, thc distance from Gweru to Victoria Falls is 50 cm.


Thc actual distance is 600 km.

(r) Find the scale of the map. giving your answer in the form I : n.
O) Calcrrlate the actual distance, in kilometres, between two towns which are 9 cm apart
o. n thc map.

t2l

12t

li} Factorise completely [r,l r2l - 91,

(b) l5a2 -l9a+6.

l2l

121

tufUrfil

r5t
l{ (e) Evaluate (i) 2{ + 20,
(ii) sl + sl.
(b) Simplify' (ar')1.

Answcr (aXi) tU

(ii)

(D) ..........,........................................
I2l

l5 The numbers in the distribution 0, 4, 7, p, g.12, 15, 16 are arranged in order of


increasing size.
)
Given that, for this distribution, the mode is 7 and the median is 9,

(e) find the value of (i) p,


(it) g,
(b) calculate the arithmetic mean of the dislribution

Arswer (a)(i)p =

'El,TT [fbrn ovcr

r59
16 A bus leaves Chinhoyi at 09 36 and arrives ar Mbare Main 6us Terminus in Harare at ll 21.

(r) Express 09 36 as the time on a 12 hour clock'

(b) How long does the journeY take?

(c) Given that the bus travels at an average speed of 20 metres per second. calculatc. in
kilometres'thedistancetravelledduringthejourney

l7 Given that P=L


'-l+?t'
(e) make x the subject of the formula,

(b) calculate, giving your answer as a decimal fraction, the value of r when P = 0,25.

Answer (a) x =

r60
Fot
Ettttitct't
U*

l8 (a) Find the inverse of the matrix (it)


(b) Given that

(i I )(;)= (T)
find the value of ,rt and the value of n.

{lxrlrw![ [Turn over

l6l
h.
E*tl
tb
19

In the diagram, PQRS is a vertical walt with PO = QR = l}m. Given that the angle of
depressiott of .S from P is 26'

(r) find
G) the vilue of r,
(ll) the value of y.

(b) C-alculate the length of RS.

I Ut" this information where necgssary I


26" = 0,90, tan 260 = O'49.J
[sin 26' =QSQ,,cos

'OUIiT

16,
Fu
E*r Fi
lj- G-rl
,b
n

The diagram shows parr df the graph of

.!r=3+?-x_x2.
(e) Write down
(i) the coordinates of the turning point of the graph,
(li) the equation of the line of symmerry of the graph.

o) By drawing a tangent, csrimate the gradienr of the giaph at rhe point (2, 3).

(c) I/sing the method of counting squares, cstimate


the area bounded by the curv!,
the -r'axis, the y-axis and rhe tine i = 2, givint your
"n*iio *it
uqu"r. .

Attswcr (aXi).......... Ill

(c)

@tIr lTurn ovcr


163
Fol
F.t*rr3l
tt* I
2l Given that

8 = the set of all real numbers,


.A = the setof rational numbers,
I = the set of irrational numbers, and
C = the set of Prfune numbers'

(a) complete the Venn diagram below, labelling the sets clearly,

(b) state to which of the sets A, I and C, the following numbers belong:

(i) 2,
(li) 0,93,

GiD \R'

Arcwer (a)

til
E!
Fr
Err*rl
,b
So1 t9 schSl
and whether or not they played
?2 A survey atmed at finding out how 750 pupils
't..Jtaed ano probabilities calculated' The tree
hockey was conducted. ihe results *"It
diagram below shows some of the results of the survey'

(t) Complete the probability tree diagram shown below'

(b) Calculate how manyof the pupils cycled to school'


(c) One pupil was selected at random'
Giving each of your answers as a oommon fraction in
it's lowest term, find the
probability that the pupil selected

(i) cycled to school and playedhockey'


(ii) did not plaY hockeY.
Answcr (a\
Plsycd
hockey

Did ttot
play
hockey

Ple)rcd
hockey

Did not
play
hockey

lTurn over
'lllt,tfwlf

165
h.
E*t
lb
a

In the diairam, MAis parallel to BN.


AXB and ldXN are straight lines.
AM =7 cm, MX = 42 qt and BN = 2 cm.

(e) Describe completely the single transformation which maps triangle AXM onro
triangle BXN.

(b) Write down the ratio of the area of LBXN to the areaof NAXM.

(c) Calculate the length of MN.

l3l

(D) ratio 3 ................. I ...........:..... lU

(c) MN = .................................. cm I3l

(DI,IT

t6
Fq
2 E*t\
n
t
Fu
&*cr\ ltlr
U*
Uy
l0

o'z468lo
In a cafe, a bun costs 90 cents and a scone costs $ 1,20. Chipo wishes to buy at least two buns
and at least one scone.
scones' write down an
(e) Given that r represents the number of buns and y the numbr of
inequalityinxandaninequalityinythatsatis$Chipo'swishes.
condition that
(b) write down another inequality in x and y satisfying the additional
Chipo has only $7 20 to sPend.
Show that it simplifies to 3r + 4y <24'

(c) Draw the three inequalities on the grid given above'


Shade the unwanted region.

tu

(c) on grid 14l

{$rvl/wfr

r6,it
ZIi'BABWE SCHOOL EXAIIINATIONS COUNCIL
in collaboratlon with
UNIVERSITY OF CAMBRIDGE LOCAL EXAMINATIONS SYNDICATE
General Certificate ol Education Ordinary Level
MATHEMATICS 4A0U2
PAPER 2
rhursday 12 NOVEMBER 1998 2 hours 30 minutes
Additional materials:
Answer paper
Geometrical instruments
Graph paper (3 sheets)
Mathematical tables
Plain paper (1 sheet)

nME 2 hours 30 minutes

INSTRUCTIONS TO CAND]DATES
Write your name, Centre number and candidate number in the spaces provided on the answer paperl
answer booklet.
Answer all questions in Section A and any three questions lrom Section B.
Write your answers and working on the separate answer paper provided.
lf you use more than one sheet ol paper, fasten the sheets together.
Electronic calculators must not be used.
Allworking must be clearly shown. lt should be done on the same sheet as the rest of the answer.
Omission ol essentialworking will result in loss of marks.
lf the degree of accuracy is not specilied in the question and if the answer is not exacl, the answer should
be given to three signilicant fQures. Answers in degrees should be given to one decimal place.

INFORilATION FOR CANOIDATES


The number of marks is given in brackets I lat the erd of each question or part question.
Mathematicaltables may be used to evaluate explicit numerical expressions.

MFK (0654)OK87689
c ZSEC t998 flurn over

169
Section A [64 marksl

Answer dl the questions in this section.

(e) Solve the equation


x-l b-3 5
3 4 -6'
-A

(b) For the expressions 4x2(x + l), &r(x + l)(.r - 3) and 6(x + l)r, find
(i) the H.C.F.,
(ii) the L.C.M.
t3l
(c) An empty-drum has a capacity of 9 kilolitres. A liquid is pumped into it at a uniform rate
of l50cm'per second..Find the time, in hours and minutes, that it will take to fill the
drum. [3] r

(e)

\D
.A
In the diagram, ABCD is a quadrilateral in which BAD = 76", ADC = 12ff and BCD = 86o.
E is a point on CD such that CBE = 36o and'ABE r2y".
Find (il BED, lrl"
(ii) the value of y. 121

170
,(b)

ln the d.iagr1[.-A B, BC and CD are three adjacent sides of a regular polygon centre O.
Given that ABC = 160", calculate
(i) the number of sides of the polygon,
I2l
(ii) cdo.
I2l
(iii) o6o.
I2l
(ivl DiB.
I2l

(e)

:l
rrl

i!
1..

+
In the diagram, O4"BC is a'parallelogram in which il =3p - q and p6'= p + 5q.
(i) Express as simply as possible, in terms of p and q,
(a) 8? tu
(b) fr.
121
(ii) Given ttratO:E =,t (8.+ q) fild rhe vatue of t.
I-M^J I2l
(b) L, M and N are three points wirh coordinates (-1, 3), (2, 5) and (- 3, - 5) respectively.
Finp (i) Nt - in column vector form, ')
.-- trl
(ii) i Nt + LM in column vecror form,
' t3l
-q>
(iii) lMNl
tzl

lTurn over

t7t
In lhis question take r to be f .

(a)

O is the centre of a circle with radius 7 cm. P and R are points on the circumference such that
POa = 9oo. Tangenrs at P and R meet at Q. Calculate the area of the shaded portion in the
diagram.
t3l
(b)

In the diagram, OPQ is a sector of a iircle cenrre O in which POg = l08o and arc pe = 33 cm.
Calculate (i) the radius of the sector, l3l
(ii) the perimeter of the secror. t2l

[In part (c) take the curved surface area of a cone to be nrl, where r is the radius of the base of rhe
cone and / is the slant height.l

(c) The ratio of the radius of a cone to its slant height is 2 : 3.


Calculate the curved:;turface area of this cone given that the slant height is 4,5 m.
t3l

.llI}til/WgS

t72

b"
Ansrrer the whole of this r;uestion on a sheet of
plain paper.
use ruler il'td u"'tPus'tt's tnily
ftt ull urttstrttt'rirttt.r tutil .sltrtrt, clettrl!, ttll tltt, t ttrstrttcrittrt lines drrtl ora.r,
(o) ( irnstrucr. on a single diagram.
(i) parallelogram ABCD in which AR 9cnr.
= /tC. = 6.5 cm antl A6C.=
(ii) the perpendicular from g kt DC..
1.5(p.
, lrl
12|.
(b) (i) Measure and wrire down rhe lengrh of rhe perpendicular from B to DC anr) hence
calculate rhe area of the parallelogr-am.
(ii) l3l
DC_ is part of a .ertain tocus. Wrire down the condition
on DC. sarisfied by all rhe points
I2l
(a)

'l'hc diagram
represenrs a trianqular rrrism.
'l'hc faccs
ABCD. BCEF an,J iOt;f arc recrangular.

Uif = CDE = gly. CED = Z7o,.,lf = 2.s cm an<l f [, = .lo cnr.


Calculare (i) zlE.
12l
(ii1 Pa
12l
(iii) 16" angle of elevation of IJ fnrm L..
t3l
.4 B

ffi
(i) I and I are square shapes. state rhe number .[ squares through which
(a) a diagonat of square zl woutd pass.
(b) a diagonal of square B woulct pasS.
l2l
(ii) A square ltoor is tired with square rircs of rhc same
size.
through l0r rires. Find the rorar, pinrber of rites Its two diagonals pass
uscd r, cover the whole floor.
t3l

lTurn over

\\a
Section B [.16 marksl
r l/l
Answer lhree qut'stions in this section.

Each question in this section carries l? marks.

(r) Given that log,,,(.r + 2l + log,,,(r + 4) = l. show that -r:'+ 6.r - 2= O. l.r I

Solve the quadratic equation rr + 6.t - 2= 0 giving v()ur ans\\'t'rs c()rrcct l() llr'() decint:rl
places. l{l
Explain brie0y whv only the positive value of .r siitislies thc cquittion

log,,,(.r + 2) + log,,,(.r + 4; = 1. lrl


(b)

In the diagram, EFG is a triangle in which EF =3 cm. I.G = 4 cm ancl/j(i=6cnl,


Calculate EFG. l1l

Arswer the whole of this question on e sheel of graph PoPel.

A school's poultry club enclosed a rectangular area of ground. They uscd part of a wall as one sidc
and 16 m of chick-mesh wire for the other three sides.

(r) If the length of each of the two sides at right angles to the wall is .r nr. show that thc arca
enclosed ii given by A = l6x -2x2. I3l

(b) Below is the table of values for the function A = l6x - 2.r2 for values of r betwccn I and 7.
Find the values of p and g. t2l

x I 2 3.5 4*5 6 7

A p 24 31.5 31,5 q l4

(c) Usingahorizontalscaleof 2cm torePresent I m and a vertical scale of 2cm to represent4 m:.
drat'the.graph of A ='16r -?t2. I4l

(d) Usc the.graph to estimate


(i) the maximum possible area thal can be enclosed and the corresponding value ol .r'. t2l
(ii) the dimensions of the recrangles which enclose an area of 28 mr. l2l

lrlg].*{a

t74

k
Ansner the whole of this question on a sheet of graph pape?.
'lhe heights
of 4-5 puiils in a cerrain class are distributed as shown in rhe rable below.

Height /r(cm) tiO<rr<90 90<A<100 100</,<ll0 ll0<ft<ll5 ll5<ft<120


Number of pupils 3 lt l5 l0 6

(a) (i) Draw the frequency polygon for this distribution, using a scale of 2 cm to
represent a
height of 5 cm on the horizontal axis and 2 cm to represent 2 pupils on
axis.
ihi vertical
lsl
(ii) Calculate rhe mean height.
I2l
\\\ 0r. 1uq\\ rs q\rrrse.\ \\ \Nr\sR. C{qrr\lre. t\e. grs\sabl\ir-1 r]tra\ rlcre 1,u1,\\s \e\r,t.r \s gxearer
than l(X) cm but less than or equal to I 15 cm. l2l

(c) 'fhree pupils are chosen at random. Calculate the probability that all their heights are less th-an
or equal to 90 cm. t3l

l0 (a) (i) Irr a shop a gas stove has a marked price of $7500. If the shop-owner gives a discount
of lllolo for cash, calculate the cash price. 121

(ii) Alrernatively. the gas stove can be bought under a Hire Purchase agreement over a period
of three years. A deposit of $15(X) is rcquired and finance charges on lhe initial balance at
the rate'of 2O"/" per irnnum simple in(ercst are added.
1 Calculate
(a) the tinance charges for the three years, l2l
(b) the Hire Purchase price. t2l

(b) Miss Mpofu drove her car for 960 km spending $350,35 on fuel costing $4,55 per litre. Calculate
the fuel consumption of the car. giving your answer in kilometres per litre, correct to three
:ea signilitant figurei. t3I
I2l
(c) $ttfi) earned simple inrerest of $406 at a rate of 14lo/" per annum. Find the number of years for
which ttre money was invested. l3l

Questions 11 and 12 arc prinled overleaf.

I2l
t2l

lTurn over

t75
ll Answer the whole of this question on a sheel of graph Paper'

(a) Using a scale of 2 cm to reprcsent 2 units on each axis draw axes for values of -t and ! irt

the ringes0 < x < l0 and - 12 < 1' < tl'


Triangl-ePQRhasverticesatthepointsl'(2,4|.0(4.4)and8(3'2).
Draw and label triangle PQR' trl
invariant and shear
(b) Triangle p re,R, is the image of rriangle PQR undcr a shcar rvith'the .l'-axis
factor - 4.
Draw and label triangle P rQ,R,- I3l

(c) (i) Drat/the line y =x - 4 and latrel it /' lrl


Triangle PzQzRzis the image of triangte PQR under a reflection in the
(ii) line /'
Draw and label triangle P 2Q 2R:' I3l

(d) Triang,le P,Q.,R, has vertices at the poinrs P,


(3.8). O: (6. 8) and Rl (4j,4)'
(i) Draw and label triangle PrQ$t' trl
(ii) Describe fully the single transformation which maps triangle PQR onto trianglc
pr0rRr. t-j I

The diagram shows rhe cross-secrion of a cylindrical glass tube into which a liquid
is pourcd'
12 side of the tube so that the surfacc of
The surface of the liquid is attracted, by surfaci tension, up the
the liquid is hemisPherical.

t
3mm

f
l7 mm

ln this question lake r to be ?.


[The volume of a sphere is ! rr3-l
and the liquid
(e) The radius of the tube and that of the hemispherical surface are equal to 3 mm
rises up the side of the cylinder to a height of l7 mm'

Calculate the volume, in cubic millimetres, of the liquid in the tube from
(i) level.4 to level 8- l3l
(ii) level B to level C, t4l

(iii) level A to level C. l2l


I,
(b) Given that the liquid has a density of 0,tt g/cm calculate the mass of the liquid in the tube from
level.A to level C. I3l

{r$1,wgfi

rre
'lypes of merks

(l) B marks are indcpcndcnt to nrethod and are usuaily awarded for an
accurate rcsult or statement.

(ri) M marks are awarded for any correct methd applied to the appropriate numbers,
cven though a numerical error rnay be involved.

(a) Once earned they cannot bc lost. ,


(b) 'l'hcy arc earned for a numerical statement which is
explicit as regards the quantity to bc found.
(c) 'fhc use of a wrong formula, wrong trigonometrical
ratio or misapplication of Pythagoras theorem is
I wrong method.

(iii) A marks are given for a numerically correct stage, for a


conect result or for an answer lying within a specified
range.

f (a) They are given only if the relevant M mark has been
earned.
(b) They are not given for a fortuitously correct
rcsult following one or morle erors in working.

(iv) MA marks imply both method and accuracy. Once eerned


they cannot be lost.

(v) In graph of drawing questions, some marks carry an


identification lettcr (e.g. Cl for drawing a curve).

(vi) The symbot t' is used to indicate correct working


following an error in computation. This is referrcd to
as a follow-through mark.

rl
{l
2l

m
3l

177
400/8I40/28IOI MATHEMATICS JUNE 1996

EXPECTFN ANSWERS

l(a) 580 (b), 0,0076 (c) 45

2(a) l2.43pm (b) 4 hr 15 min (c) 2135 or 9.35pm

3(a) 7n2 o) 2t3,

4(a) rU (b) t6 (c) lTth

5(a) 9:2 o) $l12

6(a) 5_3 o) 39 (c)

7(a) tx+5Xx-2) o) (x+5Xx-2)2

8(a) 4Ol, o) loil (c) 0and I


^r- -\
9

l0(a) 0,9208 (b) 0,2084 (c) 1,0686

o)
.,'''
\
I l(a) 3 -:,p4
l3(a) R,S O) CiM,A (c) H,S

l4(a) 124 o) 008s2

l5(a) re-2U2
,
O) -2; -l; o;

l6(a) 8 o) 13

-lt(a) 76,50 (b) 240

l8(a) 8 x l&llx6 = 14

o) p=12; g=10
l{a) 4(b)

178
20(a) 8/17 (bxi) l5 (ii) 20,1

2l(a) i-z -ro) o) irz


f)
(s 23) \-z
22 &-l
2

23(aXi) 360 (ii) 180 - 360

n il
o) 9

24(a)ti) 0 (ii) l15

(b) 23,5

25(a)(i) -l (ii) Y =-x-5

(b) (-l,6)

26(a) l/15. (b) 0 (c) 2t5

27(a'l V=kT (b) 4t3 (c) 5


P

+ZOx O) x=10; V=5Q0 \


28(a) 2x2
t

t79

\i
MARKING SCHEME : JUNE 1996 4mtm2
l. (a) (i) -5
BI
(ii) 2s
BI
liii; -f or-0,t666'
BI
(b) 2r3+3(r- l) - 2(x- l)or.quir.*1.
MI
3x-2x - 3-2 -6orequiv.
MAI
(x -) -5
AI
(c) (i) (x - 3y) (x + 3y) or equiv.
BI
t5
Bl dep on
lst Bl
(ii) x + x - 3 + 5 orequiv.soi I
or3y - (-ly) - 3 - 5 orequiv.soi .l
MI
(x-) 4ad 0-l - or -0,3333
{ AI 4

lt0l

(a) (i) ($)t02E


BI
(ii) figs 57825
Em or equiv.

MI
(f) 4s
AI
(b) (i) Conilone t )
(a) 1,8 - lforeacherror
82
(b) 8
BI
(c) 3,5,7,9 - I for each error 82
SC After B0 give SCBt for
(puhis e), or pv e seen

(ii) s
Bt/ 6
(llD
- -oreot
{or lel

rt0
3. (a) Accept answerc in diagram if clear.

(i) subtencled by equal / same chords / arcs or


BI
BC. DC

(ii) (a) 62,",


.Br
(b) 180 - (28 + 28 + 90) orequiv. MI
34r"t
AI
(b) (i) (a) Triangle with 0 - 45 t. T
Q-3G + 2
PQ - l0 t 0,2cm AI
,\
ij - 45,' t 2,'with correct
construction arcs seen cr.
/\
Q: 3(P + 2" with correct
construction ancs seen
cr
Tolerances: Accept dashed / dotted lines
Ignore wrong labels if clear
No penalty for use of lined or
graph paper

Scale drawings: ) '/r size Du C,, Cr


< '/z size Scorc 0
J marks available
(b) Ruled bisccor of his pe within tf
and t Q2 cm at least 3 cm long
BI
Ruld bisccor of his eR within tll"
and t 0,2 cm at lcast 3 cm long
BI
Correct ancs soen for both bisectors
Bl dep
on first
BI BI
(c) Complete circle cen[r his S passing
through B e and R within 0,1 cm
st
(ii) 5,2 *,0,2 cm statcd Al dep'
on Al
srt
uzl

lEl
lq
:

9 coa 45,5'"' or equiv.


Ml,
t
6,306 (csr) AI

O) qff'x2x:r,xg orequiv.
M2
50
7,1fi (cm) AI

O) 455'"'f, lr r 9 x9 orquivsoi M2
360e'

'lt x 9 r 9 r sin 45,50 orequiv. soi, M2


his(45j r r x 9 x 9)sratcd-his(t/r x 9 x 9 x sin45,S,,,)statetl MI
360

300
3,270 (cm2) AI 6

lltl

t82
t.a) (i) A PQT /// A pSR or any correcr
combinations MI
Equiangular / equal angles / same angles AI
Accept (AAA) or P is common {T - pSQ anO
PTS - PRS

( ii) PS - ti or equiv soi


54 MI
his PS stated - 4 MI
6 (cm) c.a.o. AI 5

(b) (i) (a) /- 13 \ - | for each wrong element


\23 , and/or ommission of bracket (s) 82
/-
(b)
V/3-
+ ( -7)' or cquiv vri MI

7,616 AI

(ii) Condone omitted underlines. Only exoressions


liner in f
and q' .rrf in terms J i'.rrd;;
q'
can scone

(a) -p+q+. tsw BI


r/,his (^S)orequiv
(b) v, t} MI

'ln t 6' * dl or equiv isw AI 7

u2l

,i

i:

ii
ri
j,

183
2(3 - x) - -2 - 0 soi rrerluiv. MI
(r-)4 AI
(b) -l dice for missing bracker (s)

(i) (0,4) , -l for wrong entry 82


,
(ii) 4 + 3 (5) - 5- or cquiv soi MI

I
- x)(t +
AI
(iii) (4 x)(-0) orquivsoi M2
(a) (A-) (-1,0) AI
(b) (B -) (4,0) AI
liv) (r-; -^ilrh; 1,5 or t/:
BI E

ll0l
7. (a) Tsin (180 - 135) oreouiv. M2
o-
6'
55,61t' or 5535' AI
(b) 90 - his (a) MI
034("t or N34P|E or 34PtE of N AI
(c) 7 sin 45 or equiv. MI
4,950 (km) AI
(d) (BD2-; 7' + l0' t Zx7x l0 cos 135 orcos (180- 135) MI
7' + l0' + 2x7x I0 cos 45 MI
248O0
24799

tw
\lhit24fl,q)
1',

MI
dep on first
MI
5
l5,Zl 0m) AI 5

trzl

It4
(a) F'ormula : providal purely numerical
p + (or + or -) r,G' but not t p)
r
is seen or used and r>
glY..?l_!. P
sor by conect answer.
- -2, Bl for 9 - 44 or rft- - 6,6$f
Comp. square (x + l)(2)
BI
tris Ct) t (or + or -) y'hh-il
BI
Final answer (nww but conaione pA
if correct answes rerched) 4,32 Bl; 2,32
BI 4
If wrongly or not corroctcd give Bl for both
2,317 and .4,312 seen-.

(b) (i) (V -) 25=! 2 c equiv.


!0
a MI
30 (km/h)
AI
(ii) (a -) Figs 36 or oquiv soi
MI
."-*

Eigs=16
5x60r60x60
,. equiv
Ml .4
I or 0,0333 ny's2
m AT

(iii) Figs 25 += fig 15 statcd or equiv eoi


25=his
MI

2,5 + his lJ statcd


MI
E
60

e'6 (kr/h)
AI

It5
9. (a) For tolerances and penalties refer to Qu. 12
x+yS60 BI I

(b) x22y 82 2

(c) 7t + 7,5y > 210 or equiv. MI


Correct reduction to
l4x + lSy > 420 even with no intermediate step AI

(d) x + y - 60 colroctdrawn LI

f - 'lzx conectdrawn LI
l4x + l5y - 420 correct drawn LI
Correct region identified and bounded by solid
lincsincluding y2 0 RI

(c) (i) 20 BI

(ii) his 40 x 7 + his 20 x'1,5 MI


($) 430 cao AI 3

tl2l

lE6
1

10. (a) (i) ,r x I l0 x I l0 x 345 or equiv. Ml.

20 000
13 ll0(m AI

l3lm (litres) Al ./

(ii) 2xttxfigs nxfig345+2xnx ll0x llO


or equiv M2

31,4 (mr) At6


(b) nx ll0x Il0x(345-30) MI

n0
il974 (lirrcs) AI

(c) Figs 35 xri


fig 7 MI

ll x ll0x llOx (345 - 30) x 20soi MI

his (rx ll0x ll0x(345 - 30)x20) oraluiv. MI


ffi
\-fisT /
92
4 788 (litrcJhour) AI

lr2l

/
It7

)
ll. (a) 54 BI

O) 5m - C where 2m -+ C MI
49 AI
(c) (i) 5OO-O whcreD+75

!0 r lfl)
s00 MI
t2(%) AI

(ii) 400 - 310 r lfi)


500 MI

26 (%',) Al '4

(d) !!! o Q7z


25k BI

(e) 6
x-255 BI

5
Y '34 BI

(0 (D 3 (cm) BI
(ii) 6 (cm) BI 2

u2l

ltt
t
12. Condone inaccurate or missing labels if no ambiguity.
Ignore any additional triangles. Con&ne inaccuracy up to I mm in
plouing or drawing.

Fenalties: Wrong scales: - l once.


Axes reversed:- I if not labelled.
I(o:r-uniform scale: -2 aftrr marking as
generously as pnssible.

If ruled or plain paper is used extend tolerance to 2 mm.

(a) A X corrcctly drawn with vertices at (3,2); AI


(5,2); and (5,4) |

(b) AT (X) correctly drawn with vertices at (0,6),


-
(2,6) and iZ,g) t for erch enor n ,l
[SC. After'l O give SCB I for alt correcr
coordinates seen or plotrcd) 2

(c) A P (X) correctly drawn with vertices at


(3, -2), (5, -2) and (5, -4) Pl t

(d) (i) A Q tXl conEctly drawn with vertices at


(-1, -2), (-3, -2) and (-3. -4) . Ql

(ii) Rotation.and no other tonrforrr,"tion stated Ml


Centrre (l,O) $dangle lt(P At .j

(e) x-l Et I

(f) (i) A R (X) cor€ctly drawn with vertices at


(66), (10,6) and (10,12) Ql

(ii) TUo-way str€tch and no other transformation


stated. Ml

Scale factors 2 and 3 AIAI 4

Il2l

189
400l&l N28t2 MATHEIIIATICS JUNE 1996

EXPECTED ANSWERS

l(aXi) -5 (ii) 2s (iii) -U6 (b) -5

(cXi) 15 (iD x=4: y---ll3


2(aXi) $1928 (ii) f4s

(bxi) 1,8 (b) 8 (c) 3,5,7,9 ii) (

3(aXiiXa) 62" (b) 34" (bxii) 5 to 5,4 cm


4(a) 6,307 (b) 7,148 (c) 3,285 cm2

5(aXii) 6 cm (b)(ixa)/-I3 \ (b) 7;6t6


\B)
(iD -p+q (b) l/4(p+q)
6(a) 4 (bxi) (0,4) (ii) -6
(iii) (a) (-1,0) (b) (4,0) (iv) x=lth
7(a) 55,6o (b) 034o or N34'E (c) 4,95 km (d) 15,74 km

8(a) 2,32;4,32 (bxi) 30km/h (ii) l/3om/sec (ii) 9,6 km/h


9(a) x+y<60 (b) x>zy (eXi) 20 (ii) $430
l0(aXi) 13 100 litres (ii) 31,44 m2 (b) ll 980 litres (c) 4790 litres/hr
I 1(a) s4% O) 4e (cXi) t2% (ii) 26% (d) t&tzs
1l(di) y=255; z=35 (0(i) 3cm (ii) 6 cm
12(dxii) Rotation, centre (1,0) through 180'

(e) x= I (f) two way stretch, factors 2 ard 3

l90
4@8tfi28il
NOVEMBER 199('
EXPECTED ANSWERS

I(a) h O) 12,75 (c) 0,3


2(a) xyg+x) (b) 2(7-x)(7 +x)
3(aXi) 7,4 (iiy 0,42 (b) 1,2 x lOt
4(a) t5rnr n3 (b) 4

5(a) 4.2 O) 15,75

6(a) 9t4t (b) 9t4t (c) -V2

7(aXi) 72 1ii) l5 o)0


8(a) 9:l G) 3,85

9(a) 35o G) 900 (c) 350

lO(a) lll2 O) 10105 (c) lool i2

lto) -3or0
t{al t7 o) 2Q
l3(a) .=hr-'G) k=l (c)
r
l4(a) @4 O) ll5 (c) n
l5(a) -3t5 (b) sM-s
M+l
l6(a) 360 (b) 1980 (c) 2s2
l7(a) 3 (bxi) 13 (ii) t9
-30
l8(aXi) B,I,A (ii) .I,N o) @

r9l
19(a) I (b) 72 (c) 4

20(a) 1,5 (b) 13 (c) 155

2l(aXi) 0 (ii) 33

22(aXt) 4 (ii) 3

oxi) o,w (ii) 1,602

23(a) (-2,0); (5,0)i (0,10)

ox0 -2 (ii) )r'= -2x + l0

(c) Y=2x4
24(aXt) AOB (ii) DCO or DFE O) 10,5 (c)

25(aXt) 32 (ii) 2'.165


- G) 13,3
^

ty2

I
MARKING SCHEME NOVEMBER 1996 4lm8t2

l. (a) 4(l -9) orequiv. soi MAI


-32 AI (2)

(b) In both parts condone missing bracket (s) at


end(s) of exprcssion and also condone
consistent use of other letters.

(i) (3 + x) (4 + x) or equiv. BI
(ii) k(m - n) + l(n - m) or m(k - l) + n(l - k) or equiv. MI
(k - l) (m - n) or equiv. Ar (3)

(c) 3t(r + 2) - 242 - 3t) (inner brackets are essentiat)


l.2rt or 2*t or (4r) (6t) soi or equiv. MI
F.A.3tr + 6t - 4r + 6rt or equiv c.a.o. AI
l2rt

9rt + 6t - 4f or equiv.
- AI
(3)
-Tfil
2. (a) (i) nAq or equiv. BI
e0c or n0u or equiv. BI
oer o, rde or equiv. BI
Accept answem on diagrams if clear
/\
(ii) (a) BCD - 180"' - 80"' orequiv.
/\
or BOD - 2 x 8()"' or equiv. soi MI
2 x his gCp or 360"' - tris g6p
or equiv. MI
200"' c.a.o. Alr 6
r8o-(80-4l - (80-4s)
(b) 8(}o' - 45'o) or equiv. soi. MI
35(o) Al (2)

1t2l

t93
(b) (i) (A QPT) - 6 (cm:) 82
(ii) (A PTS) - 'lz x6 or equiv. soi MI
(A QPS) - his b (i) + his '/a x6
or equiv. soi MI
9 (cm2) c.a.o.
At (4)

3. Condone comma (s) but - l once for fraction line(s), missing


brackct(s) and/or further wrong simpl i fication.
(i) (a) -l

f) *,
(o) t for cach wrong enrry or -+
(_3 -
(3 -i )
After Bo awarrt ScB ! for ht, t (3 3)
(c) -t for each wrons entry.
(_4e _11) ", (s)
(ii) (his (i) (b))
(:j)"t (e-) -8 At (t:) zt AI
award t"r(;) -
(-ji)or (e; fl : (n; _2t)
Other methods: Ml for a correct linear
equation in one variable

A l, A l for correct answers (3)


(b) (i) (a) 39, 52 or eouiv. I -
(b) 40, 50 or e{uiv. } Condone use of brackets BI
BI
(ii) (a) o Bl (b) 7 BI (4)

t94
4' ,Yry o*"T.1ltT:1-lines; ignore or no tabets irctear;
rgnore any calcutarions;
or graph paper.
no peialry r", u[;iffi;;;p",
-wrong

Uut y' marks in both cases


ane available.
(a) (i) Angle O - 60,o, ! 2,o, with correct construction
ancs
cr
Angle i - 45,o, t 2(o, with correct construction
arcs
CI
S - 610o, t 2,o,: i -
with 45,o, * z,o,

and WX - 9,0 t 0,2 cm

(ii) Ruled-bisector of his WOX


within t2b and ar
.least 3cm long from vertex
corect constructior arcs BI
seen Bl (dep. on lsBl)
(iii) A line parallel to his WX and at least
3 cm long
with tolerarrce of 2,a + o,2",
", "nd.,
ii, wxiri. MI
Correct constnrction ancs seen
A second line parallel ro his IVX AI
antl with
corr€ct il1CS Seen
At (8)

(b) (i) (vx - ) tf ",


LI
(ii)
(dep. on A t)
Points A.and B marked and labelled
at
inr,ersoction of his (a) (ii) anO
BI,
fris <al iiiil
0 for no diagram
Bt (3)
BI

IIU

!
ll
,11
iil
''l
i

t95

t
(a) Figs 264 or equiv. soi M2
IESTTs-fiCI2)
Sc. After M0A0 award ScBl for 8t seen

($) 3m c.a.o. AI (3)

(b) Figs 48 x his Figs (32 - 30) or equiv. soi


FEsT'f--
After M0 give ScBl for I year difference or
figs 48 x his figs (32 - 30) soi
($) 80 c.a.o. Al (1)

(c) (i) 18(}"'- (14O,",- 40",) or equiv. soi or


360or - (140", + l80o) - 40",) or equiv. soi MI
08(}or or NEO"d or E()") E of N AI
(ii) lt0") + 4Oo! + '/z 1180", - 140(0,) or equiv.
c 36Oo) - (l40o, - 4Uo' + 2Oo,) or equiv. MI
2&oror S6r0",1V or 60o, W of S Ar (4)

(a) 32 - 42 + 62 - 2 r 6 t4 cos or equiv soi MI


(c*e -)gil/# orequiv. MI

- 43 q 0$:9
vroT 5g AI

- 26,4lot q 26 2i'. AI (4)

(b) 5 or oouiv. AM2


Sin 28to,

7
10'6 (5) (m) Accept 10,64 from long division AI (3)

(c) (i) tlzx?i2x2x7 orequiu Accept nfor22


MI
77
7i2(m) AI
(ii) l0 x his (i) or equiv. MI
An,O (m2) c.ao.
219,9 AI (4)

196
(a) (i) r x 142 x 50 or equiv. MI
30 800 (cm') AI
30 790

(ii) 2tr, x 14 x 5O + 2* x 14 or equiv. NN

5 632 (cm') A2
30

Sc. After MOAO give Sc Bl for either '


2tt x 14 x 50 or 2n x 142 seen (6)

(b) (i) r x 14'l x 30 or equiv. MI


6 160 (cm') AI
57

(ii1 /iATf orequiu MI


33,11 (cm) AI (4)

or oqulv. MI

k ork 4k c.a.o. AI
(2)
:m

E. Condone omitted underlines and / or arows. Only expressions linear


irf a and b and in tcrms of a and / or b can scotp.
(a) (i) (BA - ) 5a - 5b or equiv. isw BI
-+
(iD (AC -) 4b - 4r or equiv. isw BT
+
(iiD (oc - ) r + 4b oi equiu isw \ BI
-+ + I
MI
(iv) (oD - ) 3/r
x his OC or equiv. soi I

?.e + 3b or equiv. c.a.o. AI


3('/c + b)
-t
(v) his OD (stated) - 5b or cquiv. soi MI
r - 2b or equiu c.a.o. A1 (7)

o) (i) (f, - ) tlzt ot 2'lze q 2,5a BI


-+
(ii) (cE -) l'/ra - 4b orequiu isw BT (2)

tn
e
L

(c) (r) Parallel


Scalar multiple of each other BI
orquiv.
(dep. on Bt)
(ii) l:2 or.ttz
BI (3)
9. (a) s15 sznlsN
82

- I each eror or omission.

.[ll; 16; t9 inthisordcrscore Blf


florcs: Condong inaccuracy of
up to I mm in
plo-oing or drawing; if the
rir"t, aL nor visible
or.if wrong plots are not ur.d. a"""p
if curve
.passes within I mm of correct point.

O) TIT 4 qir* points iogerher with his three


49t!ts for 15; zo ani zc*r*"it
---J ptotted
(-l for each error)
Corect cuwe, smooth and not grossly
P3 (2)
thick
cr @)
Fenalties: scale(s) -l once. Axis rcverse.
-l even if labelled non uniforrn-;;L, -2"fd---
ma*ing as generously as possible. Linea
or plain
paper
- extelrd drawing tolerance to 2 mm.
(i) 14, Bl (ii) 18 on 19
(c) l+3 soi Ml
Bt (2}
orO2
n m
k Ar (2)

(d) 12 x ll on equiv. Ml 33k or 0,3474


20 i9 95k AI (2)

lO. Notps: g-r?*fs and penatties refer ro question


"*
9 except axis reversed _ no penalty
if UUeUea.
Allow dc$ed or frree-hand ,ii"r.
\
(a) A ABC corecttylrawn
(b) (r) A A B, C, corroctly
AI (r)
drawn
EI
(il) Enrargerneat and no other transformation
stated
MI
Centre (6,4) q A.Bracketsessential
AI
F*tff _2
Ar g)

l9E
(c) (i) A A:BrCr correctly drawn RI
(ii) Rotation and no other transformation statcd MT
Enlargement

Centrc (4,8) AI
through 180"' orequiv. (Factor-l) AI (4)
(d) A.'B,Cr corrcctly drawn with vertices at
A..(6,7), Br (7,10'/r) and Cr (9,7) s3
-l each wrcng vertex. (3)
After S0 award ScB I for all correct co-ordinates
seen or implied even in MATRIX forrn.

(a) x24 Bl y)2 BI


After BO B0 give Sc Bl for x ) 4 and y )r 2 seen (2)
(b) 60 fiDx + 120 00Oy < I ffi (XX) or equiv. seen MI
Correct reduction to x + 2y < 24 Ar (21
(even without an intermediate step)

(c) 900x + 60Oy < 12000 orequiv.seen MI


Correct reduction to 2y + 3x 40 ( AI (2)
(even without an intermediate step)

(d) Linex-4 correctly


Liney-2 Yy, )*o LI
Linex+2y-/4 " " LI
Line3x+2y-4g " " LI
Correct region shown Rl' (4)

(dep. on Ll Ll Ll)
and solid lincs

For tolerances and penaltics refer to questidn


9 except axes reversed no penalty if labelled.

(e) xorstationwE-ggns -4x-4and I BI


y ormini-buses t.. - l0 x +2y - 24lcorrerly
\ stated BI (2t

199
12. Answcrs in table arc acceptable.

l-27 BI
rl-40 or13+hisl BI
r-40 or hism BI
P-il l
BI
j' l2l BI
r-243 BI
r-%4 BI c7)

o) (i) 'tl- BI
(iD (hof tl4x3 orequiv. soi MI
elrc
or 0J625 AI
(iii) ('/r' or equiv. soi MI
nlu q O,q:Llg AI

m
I
40p/8t&28t02
NOVEMBER 1996
FVPECTED ANSWERS

l(a) -32 OXr) (x+3Xx+4) (ii) (k-lXm-n)


(c) ort+64r
lzrJ

2(aXr)., BAD, ABC/ABE, DCBFCE


(ii) (a) 2W o) 35o

OXr) 6m2 (D 9cm2

3(aXrXa) -l (b)
(r'j) (c)
c; lil
(ir) e = -8; f=21

OXr) 39;52 (b) 4o;50

(ii) (a) 0 o) 7

4OXr) 7,9 w to 8,3 cm

5(a) $300 O) $80 (cXr) 0BO'(ii) Zff


6(a) %,40 (b) 10,65 m (cXr) Xlm (ii) AM2
7(aXr) 30 Eoo c# (ii) fi32(,frf (bxr) 616&m3 GD 33,llcm
(c) l:4
8(aXO 5a.5b (ir) 4b4r (iii) a+4b (rv) 3/4s+3b

OXr) 2rhe (ir) lka-4fr, (cXD PAMIJ.EL (ii) t:Z

nt
9OXr) 14,2 to 14,8 (ii) 18 or t9 (c) 0,2 (d) 33t95 or 0,3474

l0(bxii) Enlargement, centre (6,4) factor -2

(cXit) Rotation, centre (4,8) through l80o or enlargement, centre (4,8), factor -l
ll(a) x>4; y>2(e) x=4; y=10
l2(a) l=27; m=40; n=40; p=81; Q = l2t; r=243;t=364
ox0 3t4 (ii) etr6 (iii) 27t&

2V2
40IJ8,I40,8II MATIIEMATTCS ruNE r99'

E)(PECTED ANSWERS

l(a) 0,08 o) 5t2 (c) 0,6?5

2(a) 5,30 P,rr (b) 30

3(a) (l-2xxl+2x) (b) (rn-7Xn-5)

4(c) eo) 33

5(a) 4,7' O) ,mO

6(a) KTIE O) SQUARE (c) RHOIIIBUS

1(r) $2s o) gl16

8(a) 2v (b) l(Ilh+90y


9(a) (b)
7
'5 2zS

l0(a) Iog 6 (U) 2n (c) 2m-n

l1(a) tzg" G) 2

12(a) 1600 (bxi) 3 (ii) 4

l3(a) 156o (b) 12" i

r4(a) l2 zl\ o) ,.lu -,\


[-t o ) i \, 2)
15(a) 3 o) 14

16(a) 8 (b) t6

l7(a) 3,995xld o) 1,6 x l0o


l8(a)
(r) o) (8,U'

19(a) s* O) 4o+3
af6

fr3
20(a) 3V2

O) tOb* a; lla + llb


Zl(a) 5/8 G) str4 (c) 13t28

tl(i -l O) 3 (c) 8a

23(t) (0,0)

OXr) -2 (ir) Y-axis

(c) Rotation, 90o clockrvisQ cc,ntre (1,0)

A(i 2 G) 0,7 or {,7


25(a) ()63o or 355' (bxr) 4 (ir) 16

2U

L*
}TARKING SCHEME
JUNE 1997 4008 I2
I
l. (a) 5w + 2w : 3 + l0 or eguiv. soi
(w:) MAI
or I or I.g57
Al (2)
(b) In (ii)
-(i) and
end(s).
condone missing bracket (s) ar

(i) (a) x(x - 2)


(b) (x -4) (x - 2) BI

(ii) x(x - 2) (x - 4) Bt (2)

(iii) x-4+x Bt (l)


or equrv'
;,rift* O I\{2

Z(x - 2\
iiF]-f4) AI

F.A 2
x67,
AI (4)
) (a) 6x+x+2O+ 46:360 orequ.v.
M2
(x -) 42\ot
AI (3)
(b) Accept all answers in diagram if clear ,--,,,/n
,1
(i) (RTS-) 20,",
BI
(ii) (SftX: ) 4t6tot
BI
(iii) 180 - (46 + 90 + ZO) orequiv. soi
MI
24tot
AI (4)
(c) figs 335
ry=r,
5,3 (cm)
or et;uir.
MI

AI (2)

205
(d) (y*) k or equiv. (where k is any constant)
x-3 MI
(k*) 6 soi AI
F.A. (y : ) 6
x-3 Al (3)

(a) (i) 37 BI
(ii) e BI
(iii) 33 BI
(iv) Ad - Q++3+ 6+4+lZ+5+ 15) orequiv. soi MI
ll AI (5)

(b) (i) HnSnT orequiv. BI


(ii) (I{ n T)u (S n T or (H u S) n T or cquiv. 82 (3)

4. (a) A correct linear equation in one variable MI


(a-) 2 AI (b = ) -rlz or -0,5 AI
(b) (i) - 4 BI
(ii) y + ltlz his b (i) x or equiv. MI
y + ltlz -4x or equiv. AI
(iii) 8x + 2y -3 Bl (4t

26
(c) l'ormulu : provided purely numerical.

P+(or+or-) {q butnsl + p)is

seen or used anrl r ) I give.


--_--
Bl for P: l, Bl for q - 6l or y'q:7,3
Comp. square: (x - '/r,)':' Bl; E or E BI
v36 V5
+
his ,/n (or + or -)
.re BI
i"
l,,
i
;l
F'.A. (nww but condone PA if correct ans. rc'ached) I
it
i
t,47 Bl -l,l Bl (5) t,;
,I

If wrongly or not corrected, give B I for both 1,468 and l.l3

(a) : ) 52'o' Accept in diagram


(C6O if clear Bl (l)
(b) (BC:; r/:x6 orequiv. M2
coTffi2)
4,87 (cm) Al (3)

(c) tlt x 7 x his BC sin (63 + his 52) or equiv. MI


6
15,45 (cmr) At (2)

(d) (AD': ) 7' +6' - 2x7 x 6 cos63 orequiv. MI


6,86 AI
.ffi MI
6
6,844 (cm) Ar (4)

(e) 6 sin 63 or equiv MI


7
5,346 (cm) Al (2)

207
6. Solutions should be in a single diagranr.
Accept dotted / das'hed lines. Ignore extra lrnes or any calculations.
No penalty for lined or graph paper used.

Scale draw'n*'.=,l;1ft.i;'J J marks availablc


3,?,'"t
n
(a) (i) ZWX : construction
l20o + 2o with correct Bl
wiv : 90" f 2" with correct constructron B I

Quad. with sides i0+ 0,2 cm; 7t 0,2 cm;


8t0,2cm and xfrz: 120" +2o aru)
n
WXy:99o r 2o Q2 (4)

(ii) Circle centre his W of radius


4 + O,2 cm drawn CZ (2\

Sc. After CO give ScB I for an incomplete circle centrc


his W radius 4+ O,2 cm or for a complete circle centre
his W radius > 4,2 cm or <4,2 cm.

(iii) Bisector of his YZ ruled and at least 3 cm


long with tolerance t2" B I

Correct arcs seen B I (dcP) (2)

(b) (YZ:l l1'3 r"rnl stated A! clep. on Ql 0)


(C) Conect region shaded boun<led by solid lines R2 (2)
Sc. After RO give ScB I for region shaded but
bounded by dotted lines or for regions bountled
by his lines.
Iill

208

\
7. (a) (i) ($) 145,2 (0) BI
( ii) Figs his (i) or equiv. MI
Figs 33

M c.a.o. AI (3)

(b) (i) Fig 6 x Figs 12 or equrv. MI


($) 72 000 AI
(ii) Figs his 72 x [figs I I ]: soi M2
($) 87 120 c.a.o. AI (s)

(c) (i) Figs 18 or equiv. soi MI


FisT-
45 (cm) AI
(ii) Fig 3 x figs 12 x [fig 4l] or equiv. MI

sz!3 (m') AI (4)

8. (a) 3,0 (m); 2,5 (m); 1,2 (m) -l each error B2 (2)

(b) (i) his fig 3 stated x his figs 25 stated


x his figs 12 stated or equiv. MI
9
figs 8999 c.a.o. AI
e (l) AI
(ii) Figs 28 x figs 33 x figs 15 -
figs his b(i) or equiv. MI
I
4,860 (mr) c.a.o. A1

(iii) 2xfigs 23xfigs 15+2 xfigs


33 x figs 15 or equiv. MI

18,30 (m2) Al (7)

'-.7

209
(c) (i) his b(iii)
r BI

(ii) figs c (i) x figs 245 or equiv. Mt


2
($) 98,00 c.a.o. At (3)

9. Allow lmm tolerance at vertices of triangles and


free-hand sides. condone inaccurate or missing labels if clear.
Ignore any additional triangles.

Penalties: wrong scale(s) - r once axes reversed: no penalty if labelled


but -l if nor.

Non uniform scale : -2 often marking as


generously as possible"

Lined or plain paper : extend tolerance to 2 mm.

(a) AAcorrcctlydrawn Ar
A B correctly drawn Br (2)
(b) Translation and no other transformation stated MI
Translation vec .t
(:[ ) -, each wrong entry A2 (3)

(c) (i) Correct line drawn LI


(ii) C correctly drawn with vertices at (6, _2);
(0.8) and (8, -4) -l each wrong vertex
c2 (3)
(d) D correctly drawn DI
Two way stretch and no other transformation
stated
MI
Scale factor. -2 in x - axis direction AI
Scale factor 3 in y - axis direction Ar (4)

2to

t
10. (a) 3tlt<.xS6 soi BI
4<x(8 soi BI
(x-) 5and6 Bl Bl (4)

(b) (i) (a) Y : tl,tx +2 or lquiv. LI


(b)y20 BI
x22 BI
2y +3x < 18 orequiv. BI

y< his (i. ,) or equiv. BI


(ii) (6,0) 82
30 Bl (8)

ll. (a) (300 -145) x lfi)


--Tm-- MI
51,67 (o/o) At (2)

(b) Table with correct values


4O<xS50 50<xS60 7-5<x<80
82
73 6l 9
-l each error / ommission

7,3 6,1 1,8 82


-l each error/ ommission
(4)

(c) For tolerances and penalties refer to question


9 except -l for axes reversed even if labelled.

Rectangles with hts and centres as in table

Centre 22,5 35 45 55 67,5 77,5 90

Height 2 4,2 his 7,3 his 6,1 5 his 1,8 0,5

H3
-l eaih error (3)

2lt
(d) 15+30 30+40 4O+50 50+60
(
30x( 2 )+( 2 )+his73x ( 2 )+his6l 2 )
300

ffi+75 75+80 80 + 100


+75x( 2 )+his9x( 2 )+ l0x( 2 ) orequiv. M2

51,48 (7o\ AI (31

12. Condone omitted underlines.


Only expressions linear in a and b and in terms of
a and/or b can score.
-+
(a) (i) (oQ
-) 5b + 3a or equiv. BI
-,
(ii) (RP - ) 5b
4a - or equiv. BI
(iii) (PQ - ) 5b-a orequiv.
a - 4a + his (i) simplified BI (3t

(b) ri) k his (4a - 5b) or equiv. BI

(ii) 5b + his b (i) MI


simplified to 5 (l - k) b + 4ka c.a.o.
even without an intermediate step AI (3t

(c) h x his (5b + 3a) or equiv. BI (l)


(d) 5(l - k) b + 4ka - his (c) or equi,, MI
(k - ) tl, or 0,4286 c.a.o. AI
(h -) 'l', or 0,5714 c.a.o. AI (3)

(e) altor I or 1,333 or 4: 3


B2 (2)

212
1W8I 402812 MATHEMATICS
JUNE 1997
EXPECTED ANSWERS

I (a) I rtlt

(h)(iXa) x(x-2) (b) (x-4)tx-2)

( ii) xtx-2)(x-4) ( iii) 2


x(x-4)
2(a) 12

(b)(i) 20' ( ii) 46" (iii) 240

(c) 51 (d) !=6


x-3
(3t
3(aXi) 36 (ii) e (iii) 33 (iv) t2
(bxi) llnSrrT (ii) Tn(HUS)

4(a) a=2; (b) b:-Vz


(3) (bxi) -4
.
(ii) Y:-4x-lVz (iii) 8x*2y:-3
(lt (c) 4,19; -1,19

5(a) 52." (b) 4,872 (c) 18,71

(3)
(d) 6,844 (e) 5,346

6 14,0 cm
(2,
7(a)(i) $145,20 (ii) 44

(bxi) $72 000 (ii) $87 r20

(cXi) 4s (ii) 5760 m2

2t3
8(a) 2,7m;2,2m: l,2m

(bxi) 7128 litres (ii) 6,732m3 (iii) 18,3m2

(cXi) 4 (ii) $e8

e(b) Translation; vector


[,: )
(d) Two-way stretch; factors
-2 in x axis; -3 in y-axis

l0(a) 5and6
(bXiXa) y=ll4x + 2
(b) y >0; x22;
2y+3x<18; y<lt4x + 2
(ii) 30

I l(a) 51,67 (d) 51,4E

l2(a)(i) 5b+3a (ii) 4a-5b (iii) 5b-a

(b) k(4a-5b) (c) h(5b+3a) (d) h:417; k:317 (e)

214
4W8/4028/OI MATHEMATICS
NOVEMBER 1997
EXPECI'ED ANSWERS

l(a) (i) '3t4


t4/15 (ii) (b) 0,lgg
z(a) 6700 (b) 3,9,x lOa (c) 0,30
3(a) 650 (b) I150 (c) 330"
4(a) 72 (b) lo
5(a) -3 <n<5 (b) 1,2,3,4,s

6(a) 17,5 (b) 491,25

7(a) 75' (b) 28 (c) 56o

8(a) p=kq (t) 2or Vz(c) 240


r

e(b) 3 (c) l2O


lO(a) 30 (b) -2 or 8

I l(a) 5r(r_2xr+2) (b) (p-s)(p-x)

l2(a) 3,5 (bxi) {z;sl (ii) {3;a;5}


l3(a) - I (b) -3 (c) -5
l4(a) -2 (b) O or 317

I5(a) 24 (bxi) -t4 (ii) -r [-r -+')


( 14 -6J
L-s
I6(a) l0l2 (bxi) 23s (ii) I los

l7(aXi) 4t7 (ii) t/7 t0t2t I


(iii) o) 4ts
Ii
l8(aXi) 343 (ii) 2s 2Gi)4 II
oxi) li
ffi
20(a) 3ts (bxi) 20 (ii) 30 #i
ffr
fli
Ii

fli

2t5
2t(al 350 (b) 66,4A (c) 4.1I

22(a) -u4 (b) 6q-5


4+3q
23(a) ACE and DCB (b) 8/15 (cXi) 32 ( ii) 900

2a(a)(i) 19 (ii) 20 (iii) 2t (b) 20

(c) MATHEMATICS

25(bXi) Reflection in the line y = -/2

(ii) Enlargement of scale factor 2 with centre (0; -5)

(c) l:4

2t6
I

!'\.
M":
MARKING SCIIT]MI' NOVEMBER 1997 4U0812

t. (a) 12x+y -22 <>r equiv. 82 (-l each ermr) 2


ln both parts condone missing bracke(e) at
cnd(s) of cxpiession.

(b) (i) (cl - .5) (q + 2) or equr\ 82


(ii) (5 - p) (5 - p + 9) or eQuiv. M2
(5-p)'(14-p) orequiv. AI
Aftcr M0 SCB I for 70 - l9p + p: or equrv. seen
(c)
re
3(x-3) -2(2x + l) (3;:"x}$r equiv. MI

4x+5 or equiv. A2
(2x + t) (3 _ x)

2. (a) lOx - 3x :ZO - 3 or equiv. MAI


(x: ) 17 <>r 2 or 2,429 At2
7

(b) | - 2x - 2x + 4x: rtr cquiv. sccn Br I

4 - 8x + 8x - l6xr or equiv. seen Btl


5 - 4x - l2xr <lr equiv. B I isw (dep. Bl Bl) 3

(c) Figs 2171 x figs 12 or equiv. M2


ffi
14,5 (o/o per annum) AI
<>r l4t lt

After M0 give SCB I for figs 2175


figZ x fig I

217
3. (a) (i) 20 - 35 or equiv. s.o.i. MI
3-5
Ttlz or 7,5 (m/s'z) At 2

(ii) 20 - 30 - his (i) or equiv. MI


3-t
(t * ) 13 or 4 or 4,333 (s) AI
3

(iii) tlz
x 2 (20 + 35) or equiv. soi MI
55-(m) AI

(b) (i) ligs tZ or equiv. MI


trsT
0,08 (m) or 8 cm AI

(ii) frg 2 x (figs l5)r or equiv. M2

or his factors of 20 stated x (figs l5)'

67,5 (m') AI

2t8
-!

4. (a) Accept answers on diagram if clear

(i) (a) (OCB : ) 90 - 38 or equiv. MI

52'',' AI
/\
(b) (BAC : ) 38'o' BI
/\
(c) (BTC: ) his 52 - 38 or equiv. MI

i40' AI

(ii) No or equiv. MI
Becausc g6C : 76" and angle at the
circumfcrcnce would be 38" or equiv. AI

(b) (i) JaTF or equiv. soi M2


6,403 (cm) or 6,4Q AI
(ii) ri: of his (i) or equiv. soi MI
3,202(cm) or 3,20 A1
I

(a) (i) (Routc l) 217 + 78 + 203 or equiv. MI


498 (km) AI
(Route 2) 336 + 24O + 2O3 or equiv. MI
779 (km) AI

(ii) his (shorter route in (i)) -206


or equiv. MI
292 (km) Ai

(b) (M :) fig 5 x figs 45 + frg2 or equiv. MI


($) 24s AI

2t9
Wy'/

(c) (i) P: kA or equiv. \


cD + Even if rhe B I

(ii) 19: 5c + 2k or equiu. f ff:t:iHffi.


Condone use of other constants.

and l7:3c + 4k or equiv. Ml


2l :7c or l4k: 2g or equiv.
Ml
c:3 Al k:Z At 5

Notes: Tolerances: Accept dashed / clotted tines


Ignore wrong or missing labels.
No penalty for use of graph or lined paper.

6. (a) (i) Construction of 60 + 2" angle with


correct construction arcs seen.
Cl
Triangle with any two sides 9,5 + 0,2 cm
and and included angle of 6O + 2". I 2

(ii) Bisectorof his gAC ruled within t2c


andatleast3cmlong Bl
rl
conectconstuctionarcsofbisectorseen. Bl (clcp. on Bl) 2
),
(iii) Rectangle on the same side of BC as A
with sides his BC and his height and
angles 9O + 2"
R2

Correct construction of rectangle Al (dep. on R2) 3


i4
ir

, (U) ftt 8,0 (cm) stated


A I (dep. on t) I
and on lst B I of a (ii)
(ii) Points which are his BX from BC or equiv.
B2

t-
a

I
220
7. (a) Formula provided purely numerical
P + (or + or -) y'q but not +p is seen or used,
r
3
giveBlforp:5; B I for g: 37 or Vq : 6,082;
Bl firr r:6.
Comp sQ. (x - t/o)'t' B I [37- or
v36
.E 6
or Ji%, BI

3 4
his 5t or 6,082 or 1,013 BI
6 6

F.A. and nww but condone PA if correct ans


rcached!,ll BI -0.1tl BI
(lf wrongly or not corrected. givc B I for
both I,t147 and -0.1tl05 seen)

(b) (i) '/: x 5 x 3 sin nfrB : 4 or equiv. soi MI


n
sin ADB - JLor c.quiv. e.g. 4 x2 MI
l5 txs
,. 14'
(ADB: ) 32,2'"' or 32" l3l Ar3
For long and correct method M2AI
(ii) (BC': ) 3'+ 4' -2x3x4
cos ( 180 - his ADB) or equiv. MI
.l
45,30 AI
/----
r/his 45,31 soi M
(indep.)

6,731 (cm) AI
For long and correct method M2A2

ut I

/
w

8. (a) (i) 4O x20 - tlz x n x filz his (40 - (6 + 6))ll


or equiv. M2

492 (cm'z) Al 3

(ii) equiv.
figs his (i) x fig 2 or M I

0,0984 (m3) c.a.o. Al 2

(iii) figs his (ii) x figs 22 or equiv. MI

216,5 (kg) Al 2

(iv) 2x n x'/z his I4O - (6 + 6)l x tlt fig2


or equiv. M I

RE00 (cm':) Al 2

(b) tli xnx [r/z his (4O - (6 + 6))l' x figl-


or equiv.

61 500 soi Al

61,6 (l) Al 3

9. (a) (p: ) -8' BI

(b) The 6 given points and (-2, his p) correctly


plotted P3 (- I cach crnrr)

Smooth curve, not grossly thick passing through


colrect points (provided within (l mm)
dep. on P3 and Bl Cl 4

[Penalties: wrong scales : -l once, axes reversed;


no penalty if labelled but -l if not;

Non uniform scale : -2 afrer marking as generously


as possible

Lined or plain paper : extend tolerance to 2 mml

222
(c) Correct method clearly shown
on graph
MI
55
-1,62
A'l 2
(d) Intention to draw rhe line
(Seen as table of values,
Y: 3x _ I
ui l.u.f Z correct points
correct points on graph or
calculation
of intercepts)
MI
Correct line drawn
At2
90
(e) -1,84 BI o,5o BI
60
1,50
0,30 BI
accept in co-ordinate form

t0. (a) (i) tan 0: tlz or


equiv.
BI
180 + his 26,52 stated even in diagram
or equiv. soi
MI
206,6\o, or S26,6,o,W or 26.6,o,W of S
accepr 2O7 +27
OR 296ro34r or equiv.
Al 3
(ii) eQ :(_e) soi or tan) =
BI
6t;
\r(-6r MI
6,709
At3

223
(c) Correct method clearly shown on graph
MI
55
-1,62
A'1 2

(d) Intention to draw the line y : 3x - I


(Seen as table of values, ai least 2 correct points
correct points on graph or calculation
of intercepts)
MI
Correct line drawn
Al 2
90 60
(e) -1,84 Bl 0,50 Bl 1,50 BI
0,30

accept in co-ordinate form

10. (a) (i) tan 0: t/t or equiv. BI


l8O + his 26,57 stated even in diagram
or equiv. soi
MI
. 206,6,"' or S26,6,o,W or 26,6"W of S
accepr 2O7 +27
OR 206,",34 or equiv.
Al 3

(ii) ea) :(_e) soi or (aP) : (3) BI

3'+ (-6)2
MI

At3

223
g

(b) (i) Grad. of line QR -2 s.o.r BI


y_-4 - his2
x-4
OR.-4 - 2(4)+c or equrv. MI

!'2x'12 At 3

(ii) (0, -12) Bl -4 -l each error 3


-t

ll. (a) (i) (a- ) 6 BI (ii) (b- ) 7 BI


(iii) (c- ) 27 BI (iv) (d- ) 38 BI
(v) (e- ) 24 BI

(b) 252 - 152 orequiv. s.o.i MI


252 - 152 or eouiv. MI
T
k or O,25 AI
?[
(c) n(M n G) - 24 + 18 - (40 - I l) or equiv. soi MI
18 - his n (M n G) stated or equiv. s.o.i MI
lE - his n (M n G) or stated equiv. s.o.i MI
Q

k or 0,125
m AI

224
rz. (a) (i) x 30oN or 104 or 76 or equiv' soi
MI
ffi
AI
225'" , 78'o' and 57'"'

of size
Pie chart drawn with sectors 4
82 (-l each error)
37 1-2o: 78 + 2" md225 X2" and -l once for
missing labels)

Condonc I missing label'


,M2
i) . 104 x 300 x 2 or equiv'
1i
his ilEo (fr-is 480 - l)

SC; After MO 5 SCBI for l9J x 300 or equiv.


480 hrmso - l)
AI
c.a.o

BI
(b) (i) histogram or histograPh
MI
(ii) 12+9 +9+ 3orequiv' s.().1

AI
42 (puPils)
BI
(a) - 16'5 AccePt ( l6 yrs)
1ii i) 15,5
BI
(b) 14,5 - 15,5 AccePt (15 yrs)

225
w

4O08t40281O2 NOVEMBER 1997

EXPECTED ANSWERS

l(a) 2r+y-22 (bxi) (q-s)(q+2) (ii) (s-pXla-p)

(c) 4x-5
(2x+ lX3-x)

2(a) 2 zn (b) 5-4x-12x2 (c) 14,5

3(a)(i) 7V, (ii) 4 tn (iii) 5.s


(bxi) 0,08 (ii) 67,5
a(a)(iXa) 52 (b) 38 (c) t4 (ii) No
(bxi) 6,403 (ii) 3',202
s(aXi) 498;779 (ii) 292 (b) 24s
(cXi) P=cD + kA (ii) c=3 k=2
6(bxi) 8,4 ro 8,4 cm

. 7(a) 1,88; -0,18 (bxi) 32,2" (ii) 6,731

8(a)(i) 492 (ii) 0,0984 (iii) 216 (iv) 8800


(b) 61,6
6290,56
9(a) -8 (c) -1,55 (e) -1,8; 0,3; 1,5
l0(aXi) 206,6o (ii) 6708 (b)(i) y=2x-12 ( ii) (0,12)
(:*)
ll(aXi) 6 (ii) 7 (iii) 24 (iv) 38
(v) 24 : O) V4 (c) U8
l2(a)(ii) r3}t47g (b)(i) HISTOGRAM (ii) 42 (iiiXa) 15,5 to 16,5
(b) 14,5 to 15,5

226
MATHEMATICS: 4008/01 : 4028/01 JUNE 1998

EXPECTED ANSWERS

Qr. (aXi) l9 (ii) 1.6 x l0'l (b) 6


25

Q2. (aXi) 2,52 (ii) 36.16 (b) 60 000

Q3. v:-)'
Y: ll4
Q4. trl 24 (b) 168
7

Q5. (a) 70 (h) 250o or S7Q'W or 70'W of S

(c) 0.10" or N40"8 or 40oE of N

Q6. l9x - -5 l9x-5


(x-2X5x+ l) 5x:-9x-2

Q7. (a) 14 (b) 24


Q8. (a) 80" (b) ,[0o (c) 40" l

Qe. (a) 14 (b) 64


l

Qr0. 80
I

Ql l. (a) 285 (b) ' l3V2 or 13,5

Ql2. t= /T or s = l-7-
V4-t JIZ I

Q13. (a) 62,5Vo or 621/z% li

(b) 240,7 or 240 tno


(c) 7:9

Ql4. (a) 2m-7n (b) p=-2 or -2


3

Ql5. (aXi) -12 (ti) 27 (b) 16 and 3l

lt

227
G
I

Ql6. (a) l0 (b)(i) t44 (ii) 108

QI7. (a) 6. r r pm (b) 3 minutes 20 seconds

QI8. (a) 3m/s2 (b) 587znl or 5g,5m

Ql9.(a) d=6 (b) k= -12

Q20. (aXi) l:3 (ii) l:27 (b) 720


Q2l. (a) I 10002 (b) x=304

Q22. @) r,t76 (b) 2,352 (c) -0,222

Q23. (a) S at (6;_l)

; !,lfftl&Ifi ,;: *ii ;ii'".., ll, *,,.,,


Q24. (aXi) 6o (ii) 200 (iii) 120 (iv)
(b) 481 8

Q25. (a) t 27s (b) g2z2 (c) t4


Q26. (aXi) 7 (ii) (0;7)
(b) 38Vz or 38,5
(cXi) f (ii)!=x_7

/r''L

228
-E

I\IARKING SCHEI\IE 4008t2 JL'NE 1998

l. (a) ( i) 6-14+5 MAI (2)

-3 A1

(ii) 1-25 MAI


-2t A1 (2)

(iii) -9 + 10 or eouiv sol MAl


6

I or 0.1667 A1 (2',)
6

-l 0nce for missing bracket(s) at ends.

(b) (i) 7xy(4x * l) isw Bl

(ii) 5(l - 3c)(l + 3c) or equiv. isw 82

SC: After B0 give Bl for (l-3cXl +3c)


or (5-l5cXl *3c) or (l-3cX5+ l5c)
or 5(l-9c2) or equiv seen. (3)

teI

2. (a) Condone comma(s). In parts (a), (b) and (c) after B0 give SCBI for correct
elements with fractional line(s) or correct matrix seen followed by
further wrong simplification. Brackets are essential.

(3 i3) -l each error (3)

(b)
Gi) -1 each error. (2)

(c) 2x-5-2x-4orequiv sor M1

-2 A1 (2)

229
r
{

I
(d)
ffi,ri t)
simplified
or.equiv 82 each error (2)

SC: After B0 give scBl (.1


"l*, unsimplified t)o'
detl-s
tris
tf' tt,; ,),, G t)
OR no inverse if det A= 0.

leI

3. (a) g tan 57 or 9 sin 57' o'equiv. M1


sin 33'

13,86 (cm) AI (2) ,,

o) 9 or eouiv M2
cos 57

3.
16,52 (cm) A1 (3)

A
(c) t^nZYX: his 13.86 or equiv. Seen M1
4

his 73,9 - 57" or his 73'54t M1

16,9(") or 16o 54r cao A1 (3)

(d) th x 5 x his WY x sin 57 or eqvi. M2

6
34,il (cm') A1 (3)
tlu

230
4. (a) Fi-qs (9) sor
-
M1
18

hisfig9x I orequiv. MI
1,9 3600

I hr 23 min A1 (3)

(h) 6,5: + 92 + 2x 6,5 x 9 x cos 125 soi M1

6,52 + 92 + 2x 6,5 x 9 x cos (180 - 125) soi MI


190,4 AI
\
r/his 190,4 M1

13,80 (km) AI (5)


A
(c) BDA = (125 - 79) or 180 -(79 + 55) or equiv. soi M1

46c) soi A1

(AD :) 9 sin 79" M2


Sin his BDA

12,28 (km) A1 (5)

t1u
5. Accept answers on diagram if clear.
,\
(a) (i) : I
(QLM 65to) B1
,.i
(ii) (MQP:1 65(") B1
,\
(iii)(PYN:) 130(") 82 (4)

(b) (i) 48 + his pfif or equiv. s.o.i MI

fcfrH : ) g3{o) A1
,\
(ii) 90 - his EDF or equiv. s.o.i M1

@fre = ; 55(') AI

231
r-

(iii) fris C0r + his Dfo, or equiv. s.o.i N,t I

-(oiE=) 103"') nl r(r)

It0l
6. (a) Brackets essential.

(i) (J nL) UK or equiv. Bl

(ii) WnV or equiv. Bz

(iii) Pu(Q rr R') or equiv. 82 (5)

O) (i) Correct triangle with sides &,5 + 0.2cm.


t
6,8 0,2cm and l0 * 0,2cm I

Correct constructions arcs on at


least 2 vertices Tl
(ii) Circle centre his X with radius 4 t 0,2cm
drawn CI

(iii) Ruled bisector of his XZ, within t0,2cm


*2" and at least 3cm Bl

. Correct construction arcs seen Bl dep on lst Bl

Minor segment bounded by solid lines,


inside his triangle shaded B2 i1tzl

Notes: Accept dashed lines


Ignore wrong labels if clear
No penalty for use of General or graph paper
Scale drawings: 0 marks are available.

7. (a) Figs 12 x figs 2t5 x23A or equiv soi Ml


his (figs 12 x figs 215 x23A) + figs 12 or equiv Ml
($)19 095 Ar (3)

232
l....-
(h) (i) Figs l8o or equiv
figs 125

($)ts 120
AI
(ii) ($)6 300
BI
(iii) Figs 35 x ligs 08 900 - his b(ii)) x 3
MI
($)13 230 c.a.o AI
(ir'1
M2
36

($)7t7.50 c.a.o
AI (e)

u2t
s (a) -2 < x < l0 or equrv. . 82
After B0 give SCBI firr x ( l0 and -2 s x seer.
BI (3)
-l once tbr strict inequalities.

(b) l*hs60 BI
80t + 50b s 4 000 or equiv. BI
24t + l6b s ll24or equiv . BI (3)
(c) (i) Y = 0 and x * y = lQcorrectly drawn LI
5x + 2y : 20 correctly drawn LI
2y: xcorrectlydrawn LI
Accept dotted lines
rs\

Correct region identified and bounded


by solid lines dep on Ll Ll Ll RI

233
[Penalties: wrong scales :- I once.
Axes reversed:- 1 if not )abdled.
Non-uniform scale: - 2 after marking
as generously as possible.
If ruled or plain paper is used extend tolerance to 2mmJ

8
(ii) 16,4

SC: After B0 give SCBI for a correct


maximum value from his graPh. (6)

l12)

9. (a) ll.2(cm) justified ay ,/ffior equiv.

seen nww MAl (1)

(b) (i) 140x2xnx6orequiv M1


360

14,67 or l47s(cm)

(ii) his a + his b(i) + 6 + 9,5 or equiv Ml

41,41(cm) or 41,37 (cm)

(iii) t/2 x 9,5 x 6 or eipiv soi

or 140 x n x 6 x 6 or equiv soi


360

his17z x 9,5 x 6) + his140 x n x 6 x 6


360

72,5(cm2) AI (7)

(c) (i) his (bXii) x 15 or equiv. M1

1087,5 (cm') A1

234
(ii) 140 x 22 x 12 x 15 OR his b(i) x 15 Ml
-l
360 7

220 @m2't AI (4) l-

10. (a) (M:)6 B1

(N :) 12 B1 (2)

(b) 6 given points and his 2 other points


correctly plotted. P3 -1 each error

Correct curve. smooth and not grossly thick Cl dep Bl


Bl P3 (4)
[For Penalties see que. 8]

4
(c) 12,2 BI (l)

77
(d) -1,3 and 4,3 B1 (l)

(e) Intention of drawing line Y : x * 6 M1

-1,0 ans! 3.0 AI (2)

(f) Tangent drawn at x : 2 and


estimating diff y
diff x MI

-1,2 to -0.8 A1 (2')

u2l

I 1. (a) (U =) 14, (V -)25, (W :)7 82 -l for each error

o) 90<M<100 BI

(c) 8 given points correctly plotted ,!


P3 -l each error

Smooth curve not grossly thick passing CI


within lmm.

235
7*
),/

Penalties: Wrong scales -l once axis reversed -l even if labelled. Nonuniform scale:
-2 after marking as generously as possible lined or plain paper; exten<l
drawing tolerance to 2mm. condone non-uniform scale fronr 0 to 50.

(d) Attempt to use line M= 75 on graph

seen M1

9
67 (calves) A1

(e) his V x (his W) + I MI


80 79

hisVxftisW)+1x2 M1
80 79

5t
,t9k
AI

ttzl
12. (a) 3hp + (h-k)q = 7p * 3q - (p + 5q) or equiv. soi MI

3h:6 MI
OR h-k:-2orequiv.
h:2 AI
k:4 AI

o) (i) (a) E=2orequiv MI


PR3

his PR - 8 or eQuiv M1

4 (cm) A1

o) ()' s.o.i. MI

4
9 AI

236
_-

(ii) (a) B
Bl
(b) -2
82
After B0 give SCBI for 2

u2t

237
W

MATHEMATICS 4oo8 / 4028 / 02 JUNE 1998

EXPECTED ANSWERS

Ql. (aXi) -3 (ii) -21 ( iii) 1


6
(bXi) 7xy(ax+ l) (ii) 5(l-3c)(l +3c)

Q2' ("(:3 l?) (b)


(? i) (c)-2 1or -v" (-! i)
l6
Q3. cm
(a) 13,86 (b) 16,52 cm (c) 16,9o (d) 34,64 cm2
a4. (a) I h 23 min (b) tl,gO fm (c) l2,28km
Q5. (a)(i) 65' (ir) 65o (iii) l30o (bxi) 83' (ii) 55o (ii) 103"
a6. (a) (i) (J n L) LJK (ii) w n v' (iii) PU (Q n R)
Q7. (a) $19 095 OXi) 15 r20 (ii) $6 300 (iii) 13 230 (iv) $717,50
Q8. (a) -2 3.x S l0 ,

O) t+b < 60; 8ft+50b < 4000; 24t+l6U<1224

E
(c)(ii) 16,4
I
a9 GXr) 14,67 cm (ii) 4l,it0 cm (iii) 72,5 cm2
1
(cXi) I 087,5 cm3 (ir) 220,0 cmz
47-7
Q10. (e) 12,2 (d) -1,3 and 4,3
L

m=6, n=12, (c)

(e) -l,O
a4
and 3,0 (D -1,2 to -0,8
9
Qll. (a) U=14, Y=25, W=7 (b) 90<m< 100 (d) 67 calves (e) '5
79

Qt2. (a) h=2, k=4'OXi) 4 cm O) 4/9


(irXa) B o) -2

zl8
MATHEMATICS 4008/01 : 4028/01 NOVEMBER Ig[
EXPECTED ANSWERS

Ql. (aXi) l! (ii) l_ (b) 2, otg


28 4

Q2. (a) 0,007 316 (b) 0,0590 (c)

Q3. (a) l8'C (b) 9,6

Q4. (a) 75 (b) 1l

Q5. (a) I (b) 16


t

46. 4 Y=13
a
6-€P '+ '
Q7. (a) 56" (b) 33o

Q8. f=k (bxi) s25 (ii) 50


,' s,
ae. (a) 70" o)r 200 (c) I100
q
t

Qlo. (a) 14015 (b) r lplo2 (c) t0l0l2

Qll.(a) 1,69 (b) 1,845 (c) 1,155

Ql2. (a) I 200 m0 (b) r08

Ql3. (a) l(r-3Xr+3) (b) (3a-2X5a-3)

Qt4. (aXi) 17 (ir) ll5 or 0,2 (b) 4x2

Qts. (aXi) 7 (ir) I I o)e


Ql6. (a) 9.36 am (b) th45min (c) 126

Q17. (a) P (b) 0,5


l-2p

Ql8. (a) Ltt -6\ (b)m=8 n=l


6\-l 4t

239
Qle. (aXi) l9 (ii) l16 (b) 6,t2

Q20. (a)(i) (l;4) (ii) x=l o) -2a0,2 (c) 7,3 t0,l

Q2l. (a)
A L

(bxi) AandC
@O (ii) A (iii) B

422. (a) (b) 400

(cXi) I (ii) L
3 l5
q2,. G) Entargemcnt oentre X, Scale ftctor -2
7

o) 4:49 (c) 5,4

q24. (a) x>2 v > I


o) 90x+ lZJy <7n

u0
MARKING SCHEME 40a8rcz NOVEMBER 1998

1. (a) 4(x - 1) + 3 (2x - 3) = 2 x5 or equiv. s.o.i' M1

4x + $1 = 1O + 9 + 4 or equiv. even in fraction fortn,


so.i. M1

(x = ) 2,? or 23 or 2,3 A1 3
10 10

(bl (i) zlx + 1) or equiv. B1

(ii) All brackets are essential

24x2 lx - 3)(x + 1)3 or equiv, -1 each wrong


factor 823
(c) fig 9 or equiv. soi M1
figs 15

his rliog \x 1 orequiv.soi M1


(rffis/ 60 x 60

16h 4Omin A1 3

I
2. Accept answers in diqgram if clear.

(a) (i) laeo = 1 122tot B1

(iil 2ytot 36qto' - (76tor + 120(or + 86CI + 36(0r )


-
oi 360(0'- (76tot + l2olol + his 122tcill or equiv M1

(Y = ) '21tot A1 3

. (b) (i) soi M1


ffiotequiv
18 (sides) A1 2

(ii) 180(or - 160(0r or his-ext3 or equiv soi M1

tc0o =) 1o(or A1 2

( iii) ggtol - 1o(o) or equiv. soil M1

241
7
t.
fii
I

i
I
I ,\
t
(OBD - ) Tgtot A1

(ivl l ggtor - 15gtot or equiv soi M1


/\
(DAB '1 z$ts' A1 2

tt
Condone missing underlines. only expressions linear in p and g can score.

(al (i, (al q - 3p.or eqqiv i.s.w B1

(bl -(3p - el+p+5q or p+5q+ his (q-3p)


, or equrv M1

-2p + 6q or equiv i.s.w A1

(iil P + 59 + 3p-q = k(p + ql or equiv s.o.i. M1

(k=) 4 A1 5

(bl fil (A) brackets essential B1

(iil N tris(fr).(; - -r* equiv M1

(e)'oi A1

(f) A1

'ffi[? or from his MN stated or equiv. Ml 6

20 A1
11,18

2+2
4. 7x7 or 9Oxflx7x7 orequiv.soi M1
360

his (7 x7l - his (90 x f[ x 1 x7l or equiv M1


360

lOYz or 1O,5 (cm2) A1

(b) (i) lSOx2xIIxr = 33orequiv. rMl


360

( = 33x360x7orequiv. M1
108x2x22
(r =) 17Yz or 17,5 (cm) A1

(ii) his (17,5) x 2 + 33 or equiv soi M1

68 (cm) A1

(c) Es x 45 or equiv soi. M1

lI x his lvs x 4,51 x 4,5 or equriv. M1

42,43 tn2l A1 3

l-t
5. Notes

Accept dashed line(s); ignore wrong labels or no labels if clear; ignore-any


catculation; no penalty for use of lined paper on graph paper.

Scale drawings: > Yz size

LYz size O

butt'marks in both cases are avaitable.


(a) (il Angle B = 15O0 t 20 with correct construction
arcs c1

Parallelogram with sides 9,O+O,2cm and 5,0+ O,2cm


and angles 15O t 20 and 3O t 20 P1

243
tr
i
,,
//
Correct construction arcs for parallelogram A1
(dep on PI)3

(iit Ruled perpendicular line from his B to his DC


with tolerance 9Oo t 20 Lr2
Correct construction arcs c1
(dep. on Lll
(b) (i) 5
3,1 {cm} stated dep. on Pl A1

5
his 3,1 x 9 or equiv M1

31,5
27,9 (cm2l A1

(ii) 5
Points which are his 3,1cm from 82 2

AB or equiv.
I10t

6. (a) {i} ffiorequiv. M1

8
47,17 lcml A1

{ii } (DC = l 25 tan 27tot or equiv. M1

12,74 lcml A1

(iiil tan A = his {ii} or equiv. M2


his (i)

IA =l 1 5,10t or 1 5o7r c.a.o A1

(b, (it (A=l 3 (B=l 5 B1 B1

24
(iil (no. of squares along diag. =151 s.o.l B1

51
2
or €qu'' r. M1

2601 A1

7. (a) Logro (x + 2)(x+4) =l s.o.r M1

(x + 2)(x+4) = 1O(rr seen M1

Correct simPlification to

x2 + 6x -2 = O nww
Formula provided purely numer'cal

p{or + or-l/qbutnot t P
f

is seen or used give


4
81 forP = -6, Bl for I = 44 or y'q = 6,633
Blforr=2
Comp squara: (x + 3)er 81 11 ory'l1 B1

I
I
B1
his (1) +

Final answers (nww but condone PA


if correct answers reached)

O,32 or -6,32 81 81

sben,l
ISc. After Bo B0 give Sc 81 for both 0,3166 ald 6,317
5

(bl 62 = 32 + 42' 2x3x4 cos6orequiv' s'o'' M1

M1
cos a= or equiv.
t;tr;t

245
w

I Cos@ = -l 1 or-0,48 Al for 4008 onty


24

(A = ) 117, (3lro' or 1170 111 A1 4

112t
8. t Notes: cfndone inaccuracy of up to 1mm in protting or drawing; if plots
arb not visible or if wrong plots are not used accept if curve
passes within l rnm of correct point.

P.enalties: wrong scale(sl -1 once, axes Feversed -l if not labelled, non


,uniform scale -2 atter marking as generously as possible. Lined or
plain paper extend tolerance to 2mm.

(al 16x - 2x2 justified by dimensions x and 16 - 2x seen 82 z

bt (P=l 14 lq=l 24 81812


(cl .The 4 given points together with his 2 points correctty
Plofted p3 (-1 each error)

smooth curve and not grossty thick passing through correct

points (provided within lmm) Cl dep. on p3OBl 4

(dl (i, 32,2lmzl B1 (x=4) B1


31,8

(iil 5by 4 B1 7 by 11,0 B1


5,3 5,O 2,5 10,6

112t
9. Notes: For tolerances and penalties see question 8 except for axes
reversed - 1 even if labelled.
(al (i) Points 85;-95, 105, 1 12,5, 117,5 s.o.i 82 -1 each error
Points (his 85,31 {his 95,1 1}
where 8O < his 85 < 90 etc. correctly
plotted -1 each error P2

correct polygon, joined by straight lines, dep. on 82 pZ L1

Condone non-uniform scale from O - 85, an open polygon


and extra wrong graphs.

(i.l 8o + !xlx3 + !x) + 1 (x)xl 1 + loo + r 1 oxt s + 1 1 o + 1 1 sxt o+ 1 I5+ 1 2(}116

45

or equtv. M1

104,6(cmlc.a.o A1 7
(bl 15 + 1O or equiv. M1
45

5t. or 0,5556 A1
9k

(c) 3x2xl.orequiv M2
45 44 43
k or O,OOOI A1
1 41 90K

10. (a) (il figs (1OO - 181 x figs 75 or equiv. M1

($) 6 150 A1

(ii I (al figs (7 5OO - 1 5OO) x tig 2 x fig 3 or equiv. M1

($l 3 600 A1

(b) (7 5OO - 1 5OOl + 1 SOO + his (iiXal or


equiv. M1

($l 111OO c.a..o Al 6

2qt
W :-rE
1
Y;

(b) ties (8495)s.o.i. M1

fig 96 or equlv
his ris:
(ffi) M1

12,S(km/l) c.a.o A1 3

(cl Tx fig 8 x fig 145 = fig 406 or equiv. s'o'i M1

figs 4O6 or equiv. M1


figs 8 x figs 145

3,5 (Years) A1 3

tl2l
Allow l mm tolerance of vertices of triangles and free hand sides. Condone
inaccurate or missing labels if no ambiquity. lgnore any additional triangles. For
penalties. see question 8.

(a) A PGIR conec{lY drawn

(b) A P,Q,R, conectly drawn with vertices at

12, -41;,,4,'12) and (3, -10) -1 each error ss/ g

(cl (il Line y = x-4 correctly drawn and labelled L1

(ai) L'P, O, R, correctly drawn' with vertices at ng/ +

18, -21; (8, Ol and (6, -1) -1 each error

(d) (i) A PO'R. conectlY drawn T1

{ ii} Two way strech and no other transformation


stated M1

After MO give SCB1 for a stretch with the descriptions


below, scale factor 2in y - axis (direction)

or x - axis invariant A1

28
,I
-!Ilr

Scale 114 x - axis (directionl A1

ory-axisinvariant.
lJ2t
12. (al (il 17 - g s.o.i. B1

(iil IIx3x3x 117-31 orequivsoi M1

396 (mm3) A1

[I x 32 x 3 or equiv. s.o.i M1

1x 4 x [I x 33 or eguiv. s.o.i. M1
23
lI x 32 x 3 - 1 x 4 x II x 33 or equiv. M1
23
28,29(mm3) A1

(iiil his 396 + his 28,29 or equiv. M1

424,3 (mm3) c.a.o. A1

his 424,3 x 0,8 or eqgiv. M1


i'
his 424.3 x O.8 or equiv. M1
1000 . )),

0,3394 (g) c.a.o l:


in;;f
,.

.
A1
.t
i-r
v;
T MATHEIT{AIICS a008/02: 4O28l()2 TYoVEMBER t99E
*/
r{

QI. (a) 23 0)(i) 2(x+ l) (bxii) 24x2(x-3Xx+ l)3


l0

(c) 16 hours 40 minutes.

Q2. (a)(i) 122" (ii) 21.

O)G)
,I
tg (ii) l0o (iir) 70" (iv) ZA"

Q3. (aXi) -3p+q (b) -2p+6q (ii) 4

cxD (e) GD (e) (iii) I r,2o


44. (a) 10,5 cm2 OXi) lZ,5 cm (ii) 6g cm (c) 42,43m2

Q5. OXi) 3,2J cm; Zg,tS m2


(ii) Points which are 3,?S from AB

46. cm (ii) tZ,7M


(a)(i) 47,17 (iii) lj,l o or t|"]t
(bXrXa) 3 O) 5 (ii) 260l

Q7. (a) 0,32; 4,32 togarithn of negative numbcr is not defirrd

O) 1t7,3. or tt7"i7,
aE. O) p=14; j=24
'(O(0 31,8-32,2 . x=4

(ii) 5,3-5,5 by 5,(L5,4 ; 2,5-2,7 by l0,Gll,2


49. cm .(b) 5 (c)
(aXir) 1O4,6 t
9 14190

QlO. (aXi) So tso (iiXa) $3 600 O) lll0o


O) l2;S nril (c) 3,5 years

2so
\
I

fi

Ql l. (b) Pr (2,-4\ Qr(4,-12) Rt(3,-10)

(c) Pz(8,-2) Qe(8,0) Rr(6,-l)

(d) Two way stretch, scale factor 2 in y-axis and scale fa'ctor
1,5 in the x-axis.

Ql2. (aXi) 396mm3 (ii) 28,29mm3 (iii) 424,3 mm'

(b) 0,3394g

You might also like